You are on page 1of 111

Engineering Mathematics

EE Correlation 1 (Mathematics)

Module 1
(Engineering Mathematics)

Prepared by:
JAY T. OLIVEROS, REE, RME
Instructor 1

JAY T. OLIVEROS, REE, RME


Engineering Mathematics

EE Correlation 1 (Mathematics)
Module 1
(Engineering Mathematics)

INTRODUCTION: The series of Problems and Exams on this module deals with a review of all
Basic Mathematics subjects in preparation for Mathematics subject on the REE Board Exam.

TOPIC OVERVIEW: For the whole semester, the topic covered is a review of all the formulas and
principles of all Mathematics subject that covered in the REE Board Exam. To Improve the logical
thinking in solving problems by using examples based on real Board exam problems, and to solve
the REE Board problem using modern engineering techniques to answer the problem correctly.

INTENDED LEARNING OUTCOMES


At the End of this Module, Every student is expected to:

• Familiarize and identify the subject’s formulas and principles in solving problems by using
examples based on real board exam problems.
• Analyze and examine the real board exam problem logically and correctly.
• Solve the problems using modern engineering techniques to pass the board exam.

JAY T. OLIVEROS, REE, RME


Engineering Mathematics

Sample Registered Electrical Engineers Pre-Board Examination

1. Find the harmonic mean between numbers 3/8 and 4.


Solution:
2𝑎𝑏
𝐻𝑀 =
𝑎+𝑏
3
2( )(4)
8
𝐻𝑀 = 3
+4
8

𝟐𝟒
𝐻𝑀 =
𝟓

2. The terms of a sum may be grouped in any manner without affecting the result. This is law
known as:
Ans: Associative Law

3. A number is divided into two parts such that when the greater part is divided by smaller
part, the quotient is 3, and the remainder is 5. Find the smaller number if the sum of the
two numbers is 37.
Solution:
Let x = smaller part
37-x = greater part

37 − 𝑥 5
=3+
𝑥 𝑥
37-x = 3x + 5
4x = 32
X=8
4. Mary was four times as old as Lea ten years ago. If she is now twice as old as Lea, how
old is Mary?
Solution:
Let x be the present age of Lea and y be the present age of Mary
x = 2y eq. 1

10 Years Ago
X – 10 = 4(y – 10)
X – 4y + 30 = 0 eq. 2

Substitute x in eq. 1 to eq. 2


X – 4y + 30 = 0
2y – 4y + 30 = 0
-2y = -30
y = 15
Mary’s Age = 2y
= 2(15)
= 30

JAY T. OLIVEROS, REE, RME


Engineering Mathematics

5. The sum of three succeeding odd integers is 75. The largest integer is
Solution:
Let x = 1st integer
X + 2 = 2nd integer
X + 4 = 3rd integer

X + (x + 2) + (x + 4) = 75
3x = 75 – 6
X = 23
X + 2 = 23 + 2 = 25
X + 4 = 23 + 2 = 27 largest integer

6. A ship propelled to move at 25 mi/hr in still water, travels 4.2 miles upstream in the same
time that it can travel 5.8 miles downstream. Find the speed of the stream.
Solution:
𝑑𝑖𝑠. 𝑢𝑝𝑠𝑡𝑟𝑒𝑎𝑚 𝑑𝑖𝑠. 𝑑𝑜𝑤𝑛𝑠𝑡𝑟𝑒𝑎𝑚
=
𝑠𝑝. 𝑢𝑝𝑠𝑡𝑟𝑒𝑎𝑚 𝑠𝑝. 𝑑𝑜𝑤𝑛𝑠𝑡𝑟𝑒𝑎𝑚
Let x be the speed of the stream
4.2 5.8
=
25−𝑥 25+𝑥

4.2(25 + x) = 5.8 (25 – x)


4.2x + 5.8x = 145 – 105
10x = 40
X = 4 𝒎𝒊⁄𝒉𝒓

7. Jose’s rate of doing work three times as fast as Bong. On giving day Jose and Bong work
together for 4 hours then Bong was called away and Jose finishes the rest of the job in2
hours. How long would it take Bong to do the complete job alone?
Solution:
Let x = no. of hours Bong complete the
job alone
1
= Boy’s rate of doing work
𝑥
1
3 ( ) = Jose’s rate of doing the job
𝑥

1 3 3
Then; ( + ) 4 + ( ) 2 = 1
𝑥 𝑥 𝑥

4 12 6
+ + =1
𝑥 𝑥 𝑥

X = 22 hrs.
8. The length of the rectangle is 3 times its width. If the width of the rectangle is 5 inches,
what is the rectangle’s area in square inches?
Solution:
L = 3W A=LxW
L = 3(5) A = 15 x 5

JAY T. OLIVEROS, REE, RME


Engineering Mathematics

L = 15 A = 75 in²

9. For all x>2, (2x²+2x-12) / (x-2) simplifies to


Solution:
Factor 2x²+2x-12= 2(x² + x – 6)
2(𝑥+3)(𝑥−2)
= (𝑥−2)
= 𝟐(𝒙 + 𝟑)

10. If the hypotenuse of the right triangle is 10 inches long and one of its legs is 5 inches long,
how long is the other leg?
Solution:
c² = a² + b²
10² = 5² + b²
100 = 25 + b²
b² = 75
b = 5√𝟑

11. The standard (x, y) coordinate plane, the graph of (x+3)² + (y+5)² =16 is a circle. What is
the circumference of the circle, expressed in coordinate units?
Solution:
(x - h)² + (y - k)² = r²
(x + 3)² + (y + 5)² = 16
r=4
C = 2 𝜋𝑟
C = 2 𝜋(4)
C= 8 𝝅

12. How many solutions are there to the equation x²-7=0?


Note: x² -7 = 0 is a polynomial of degree 2 and the degree tells us the number of solutions.
Ans.: 2

13. A circle with center (4, -5) is tangent to y-axis in the standard (x, y) coordinate plane. What
is the radius of this circle?
Note: Since the circle is tangent to y – axis, just count the x distance from the origin (0,
0), so the answer is 4.

14. Angle A is an acute angle and sin (A) = 11/14. What is the value of cos (A)?

A
a = 5√3 C = 14
C B
b = 11
Solution:
11 𝑎𝑑𝑗
Sin (A) = Cos (A) =
14 ℎ𝑦𝑝

𝟓√𝟑
14² = a² + 11² cos (A) =
𝟏𝟒

a² = 196 – 121 −> a = 5√3

JAY T. OLIVEROS, REE, RME


Engineering Mathematics

15. What are the values of a and b, if any, where –a|b+4|>0?


Note: For –a ꞁb + 4ꞁ, -a must be positive and ꞁb + 4ꞁ must also be positive. So, a must be
negative and b+4 not equal to 0 or b not equal to -4.
Ans.: a < 0 and b ≠ -4

16. In a shipment of televisions, 1/50 of the televisions are defective. What is the ratio of
defective televisions?
Solution:
1
𝑥
50 = ( 1 ) (50)
49 50 49
𝑥
50
𝟏
=
𝟒𝟗

17. Which of the following is divisible (with no remainder) by 4?


Ans.: 510056

18. A particle travels 1 x 10⁻⁶ meters per second in a straight line for 5 x 10⁻⁶ seconds. How
many meters has it traveled?
Solution:
d = ts
d = (5 x 10⁻⁶) (1 x 10⁻⁶)
d=5m

19. The length of sides AB and AC in the triangle below are equal. What is the measure of
angle A if angle C is 70°?
Solution:
AB = AC
C=B
 A = 180 – (70 + 70)
 A = 40°

20. ABC is an equilateral triangle. AH is perpendicular to BC and has a length of 2√3. What
are the areas in square inches, of triangle ∆ ABC?
Solution:
Let s be the length of the side of the equilateral triangle
𝑠
s² = ( )² + ℎ²
2
𝑠
s² = ( )² + ( 2√3 )²
2
𝑠2
s² = + (4)(3)
4
4s² = s² + (16) (3)
3s² = (16) (3)
s² = 16
s=4

JAY T. OLIVEROS, REE, RME


Engineering Mathematics

1
A = 𝑏ℎ
2
1
A= (4)( 2√3 )
2
A = 4 √𝟑

21. Find the value of k in the quadratic equation (2k + 2) x2 + (4-k) x + k – 2 = 0 so that roots
are reciprocal of each other.
Solution:
1
Let x, be the roots
𝑥
Then,
1 𝐶
X ( ) = product of roots formula
𝑥 𝐴
𝑘−2
1=
2𝑘 + 2
k = -4

22. Solve for x if 8y = 3X-11


Solution:
8y = 3x – 11
−3𝑥 −8𝑦−11
` =
−3 −3
𝟖𝒚+𝟏𝟏
X=
𝟑

23. When graphed in (x, y) coordinate plane, at what point do the lines 2x+3y=5 and x=-2
intersect?
Solution:
2x + 3y = 5
2(-2) + 3y = 5
-4 + 3y = 5
3y = 5 + 4
3𝑦 9
=
3 3
Y=3
Ans.: (-2, 3)

24. The area of a trapezoid is 0.5h (b1 + b2), where h is the altitude, and b1 and b2 are the
length of the parallel bases. If the trapezoid has an altitude of 15 inches, an area of 105
square inches, and one of the bases 22 inches, what is the perimeter, in inches of the
trapezoid?
Solution:
A = 0.5h (b₁ + b₂)
105 = (0.5) (15) (22 + b₂)
b₂ = -8
Note: base of trapezoid cannot be negative, therefore no trapezoid exist with the given
dimension.

25. If you drove at average speed of 66 miles per hour, what distance in miles, did you drive
in 99 minutes?
Solution:
99 min x 1 ℎ𝑟⁄60 𝑚𝑖𝑛 = 1.65 hr.
66 mi/hr. x 1.65 hr. = 108.9 mi

JAY T. OLIVEROS, REE, RME


Engineering Mathematics

26. If x and y are any real numbers such that 0 < x < 2 < y, which of these must be true?
Solution:
Since both x and y are positive, then the inequality x<2<y may be written as
(1/x > ½ > 1/y)xy
y > (xy)/2 > x or
x < ( xy)/2 < y

27. In the right triangle ABC below, what is the cosine of angle A if the opposite side is 3 and
adjacent side is 4?

Solution:
A

5
3

C 4 B

c² = a² + b²
c = √𝑎2 + 𝑏 2
c = √32 + 42
c=5
𝑎𝑑𝑗 𝟑
cos(A) = =
ℎ𝑦𝑝 𝟓

28. In a triangle ABC with segment BD, B is on AD,  BAC and  ACB measures 26° and
131° respectively. What is the measure of  CBD?
Solution:
 ABC = 180 – (131° + 26°)
ABC = 23°
ABC and  CBD are supplementary so,
CBD = 180° - 23° = 157°

29. The total surface are of all six sides of the rectangular box below is equal to 128 square
inches. What is x in inches?
Solution:
Total area of front and back faces = 2(4x)
Total area of left and right faces = 2(4x)
Total area of top and bottom = 2(16)
Total area of all 6 faces =
2(4x) +2(4x) + 2(16) = 128
16x = 128-32
X = 6 inches

30. ABC is a right triangle. ABDE is a square of are 200 square inches and BCGF is a square
of 100 square inches. What is the length, in inches, of AC?

JAY T. OLIVEROS, REE, RME


Engineering Mathematics

E D

A B F

C
G
Solution:

AB = √200 = 10√2 inches

BC = √100 = 10 𝑖𝑛𝑐ℎ𝑒𝑠

AC² = (10√2)² + (10)²


AC² = 300

AC = 10√𝟑
31. What is the slope of the line: 4x = -3y + 8?
Solution:
4x = -3y + 8
y = mx + b
3𝑦 −4𝑥 + 8
=
3 3
4𝑥 8
y=− +
3 3
𝟒
Slope = -
𝟑

32. Which of the following is equal to √45 ?


Solution: √45 = 𝟑√𝟓

33. What is the smallest value of x that satisfies the equation: x (x+4) = -3.
Solution:
X(x +4) = -3
By factorization
X² + 4x + 3 =0
X² + 2x + 2x + 3 =0
3
X(x + 2) + 2(x + ) = 0
2
X+2=0 x + 3=0
X = -2 x = -3
34. A group of 7 friends are having a lunch together. Each person eats at least ¾ of a pizza.
What is the smallest number of whole pizzas needed for lunch?
Solution:
If each person eats at least ¾ of a pizza then, 7 persons will eat at least:
7 x ¾ = 5.25
= 6 pizzas

JAY T. OLIVEROS, REE, RME


Engineering Mathematics

35. There are n students in a school. If r % among the students are 12 years or younger, which
of the following expressions represents the number of students who are older than 12?
Solution:
If r% are 12 years or younger then (100 – r %) are older than 12. Hence the # of students
who are older than 12 is given by n (100 – r %) = n (100 – r)/100

36. The measures of angles A, B and C of a triangle are in the ratio 3:4:5. What is the measure,
in degrees, of the largest angle?
Solution:
3:4:5
3x + 4x + 5X = 180
12X = 180
X = 15
3(15) = 45
4(15) = 60
5(15) =75 – largest angle

37. If x + 4y = 5 and 5x + 6y = 7, then 3x = 5y =?


Solution:
X + 4y = 5
X = 5 -4y
By substitution
5x + 6y = 7
5(5 – 4y) + 6y = 7
25 -20y + 6y = 7
-14 y = -18
9
y=
7

x = 5 – 4y
9
x = 5 – 4( )
7
36
x=5-
7
1
x=-
7
1 9
Then, 3x + 5y = 3(- ) + 5( ) = 6
7 7

38. For all real number x, the minimum value of 1 + 2cos(4x) is


Solution:
The range of values of cos(4x) is as follows:
-1 ≤ cos(4x) ≤ 1
Multiply all terms by 2
-2 ≤ 2cos (4x) ≤ 2
Add +1 to all terms of inequality and simplify
-2 + 1 ≤ 2cos (4x) + 1 ≤ 2 + 1
-1 ≤ 2cos (4x) + 1 ≤ 3
The minimum value of 1 + 2cos (4x) is
-1.

39. What is the largest possible product for 2 odd integers whose sum is equal to 32?

JAY T. OLIVEROS, REE, RME


Engineering Mathematics

Solution:
Two add numbers may be written as
2n + 1 and 2k + 1 where n and k are positive integers
2n + 1 + 2k + 1 = 32
n + k = 15 or k = 15 – n
P = (2n + 1) (2k + 1)
P = (2n + 1) (2 (15-n) + 1)
P = 4n² + 60n + 31
15 15 2
P = 4(n - )² + 4 ( ) + 31
2 2
15
The product P is maximum for n = = 7.5. However, n must be an integer and therefor P
2
is maximum for integer value of n equal to 7 or 8 and both their values P = 255.

40. If (a + b) ² = 25 and (a – b)² = 45, then a² + b² = ?


Solution:
Expand
(a + b)² = 25
a² + b² + 2ab = 25

(a – b)² = 45
a² + b² - 2ab = 45

Add and simplify


a² + b² +2ab + a² + b² - 2ab = 25 + 45
2a² + 2b² = 70
a² + b² = 35

41. If a = 3, then 2 / (1/7 + 1/a) =?


Solution:
2 𝟐𝟏
=
1 1 𝟓
( + )
7 3

42. A company makes a profit equal to 25% of its sales. The profit is shared equally among
the 4 owners of the company. If the company generates sales of $5,000,000, how much
money does each one of the owners get?
Solution:
$ 5,000 x 25% = 1,250,000
$ 1,250,000 /4 = $ 312, 500

43. If the expression x³ + 2hx – 2 is equal to 6 when x = -2, what is the value of h?
Solution:
x³ + 2xh -2 = 6
(-2)³ + 2(-2)h -2 = 6
-8 – 4h -2 = 6
-4h = 6 + 10
h = -4

44. If -3/(a-3) = 3(a + 2), then a = ?


Solution:

JAY T. OLIVEROS, REE, RME


Engineering Mathematics

Multiply both terms by (a – 3) (a + 2)


−3 3
(a – 3) (a + 2)[ ] = (a – 3) (a + 2) [
(𝑎−3 ) (𝑎+2)
-3(a + 2) = 3(a – 3)
-3a – 6 = 3a – a
-3a – 3a = -9 + 6
−6a −3
=
−6 −6
𝟏
a=
𝟐
45. Which integer is nearest to √2100/√7 ?
Solution:
√2100
= 10√3 = 𝟏𝟕. 𝟑𝟐
`√7

46. The two legs of a right triangle measures 6 and 8 inches respectively. What is the area of
the circle that contains all the vertices of the triangle?
Solution:

8
88
6

c² = 6² + 8²
c² = 100
c = 10
diameter = 10
radius = 5
𝐴 = 𝜋𝑟 2
𝐴 = 𝜋(5)2
𝐴 = 𝟐𝟓𝝅

47. X and Y are accurate angle such that tany = cotx. What is the sum in degrees, of the
measure of the angles X and Y?
Solution:
tan y = cot x
1
tan y =
𝑡𝑎𝑛𝑥
tanytanx = 1
𝑡𝑎𝑛𝑥+𝑡𝑎𝑛𝑦
tan (x + y) =
(1−𝑡𝑎𝑛𝑥)(𝑡𝑎𝑛𝑦)
tanytanx = 1, tan(x + y) is undefined therefore x + y = 90°
48. What is the value of adjacent side if the opposite side is 1 inch and the 2 other angles of
the right triangle is 30° and 60°?
Solution:
1
tan (60) =
𝑎

JAY T. OLIVEROS, REE, RME


Engineering Mathematics

1 𝟏
a= =
tan(60) √𝟑

49. Which of the lines below is not parallel to the line 6x-2y = 10?
Solution:
y = mx + b
-2y = -6x + 10
y = 3x – 5
slope = 3
For all the choices only 3x + y = 7 have a slope not equal to 3:
3x + y = 7
y = -3x + 7
slope = -3
Therefore, this line is not parallel to the given line.

50. For what value of k the equation below has no value of x: 2x + 3 = x - 2kx – 5?
Solution:
2x + 3 = x – 2kx- 5
X + 2kx = -8
X (1 + 2k) = -8
−𝟖
X=
(𝟏+𝟐𝒌)
The solution x is real # and therefore the given equation has a solution except
when 1 + 2k = 0. A division by zero is not allowed so given equation has no solution if 1 +
2k = 0 or k = -1/2 or -0.5.

√x−2
51. Find the value of x which will satisfy the equation = 1?
√x
Solution:
Multiply both sides by √𝑥
2
√𝑥 (√𝑥 − = 1) √𝑥
√𝑥
X – 2 = √𝑥
Square both sides
x² - 5x + 4 = 0
(x – 4) (x – 1) = 0
X=4 x=1
Only x = 4 satisfy the original equation, other root x = 1 is extraneous.

52. Find the geometric mean between the terms -4 and -9.
Solution:
X = √ 𝑎𝑏

X = √( −4)(−9)
X = √36
X=6

53. What is the average value of 7/8 and 3/4?


Solution:
7 3 13 𝟏𝟑
+ = ÷2=
8 4 8 𝟏𝟔

JAY T. OLIVEROS, REE, RME


Engineering Mathematics

54. A solution is made of water and pure acid. If 75% of the solution is water, how many liters
of pure acid are in 20 liters of this solution?
Solution:
If 75 % of the solution is water then
100% - 75% = 25% acid
20L (25%) = 5 Liters pure acid

55. The diagonal of a square has a measure of 12 inches. What is the perimeter, in inches of
this square?
Solution:
s² + s² = 12²
2s² = 144
s² = 72
s = 6√2
P = 4s
P = 4(6√2)
P = 24 √𝟐

56. The right triangle ABC, C is a right angle and the measure of angle B is 60°. If BC is 20
inches long, then how long is AC?
Solution:
𝑎
tan (60) =
20
a = 20 tan 60
a = 20√𝟑

57. If x = 2.0001, which of the following expression has the largest value?
Solution:
The expression 𝟐⁄(𝐱 − 𝟐) has a denominator equal to x – 2 = 2.0001 – 2 = 0.0001. A
small denominator leads to a larger value

58. In the rectangle ABCD, the measure of the length AD is 3 times the measure of the width
AB. What is the slope of the line segment BD?
Solution:

B C

3x ?

A x D

−𝐶𝐷 −𝐴𝐵 −𝐴𝐵


Slope of BD = = = (3)(𝐴𝐵)
𝐴𝐷 𝐴𝐷

−3𝑥 −3𝑥 −3𝑥


= = =
𝑥 𝑥 3(3𝑥)

JAY T. OLIVEROS, REE, RME


Engineering Mathematics

𝟏
=-
𝟑

59. What is the product of the two real solution of the equation: 2x = 3-x²?
Solution:
x² + 2x – 3 = 0

a = 1 b = 2 c = -3
𝑐 −3
In finding the product = = = −𝟑
𝑎 1

60. The ratio of the circumference of any circle to the diameter of the circle is:
Ans.: Irrational Number

61. Find the sum and product of roots of the equation x³ + 2x² - 23x -60 = 0.
Solution:
x³ + 2x² - 23x – 60 = 0
factor
(x – 3) (x² + x – 20)
(x – 3) (x – 4) (x + 5)
Sum = (-3 + -4 + 5) = -2
Product = (-3) (-4) (5) = 60

62. The ratio of 3 numbers is 2:5:7. if 7 is subtracted from the second number, the resulting
numbers form an arithmetic progression. Determine the smallest of the three numbers.
Solution:
Let 2x, 5x, 7x be the numbers, 2x being the smallest
(5x – 7) – 2x = 7x – (5x – 7)
3x – 7 = 2x + 7
3x – 2x = 7 + 7
X = 14

Smallest number = 2x = 2(14) = 28

63. Determine the sum of the first 4 terms of the sequence whose general term is given by 3ⁿ
- 2.
Solution:
S = (3' – 2) + (3² -2) + (3³ - 2) + (3⁴ - 2)
S = 112

64. Find the sum of all positive integers between 84 and 719 which are exactly divisible by 5.
Solution:
The sequence is 85, 90, 95…715
a₁ = 85 an = 715 d=5
solving for n
an = a₁ + (n – 1) d
715 = 85 + (n – 1) 5
n= 127
𝑛
s = (𝑎₁ + an)
2
127
s= (85 + 715)
2

JAY T. OLIVEROS, REE, RME


Engineering Mathematics

s = 50, 800

65. If 3logx – logy = 0, express in terms of x.


Solution:
3logx – logy = 0
3logx = logy
logx³ = logy
x³ = y

66. In a certain A.P. the first, fourth and eight terms are themselves form a geometric
progression. What is the common ratio of the G.P.?
Solution:
Let a = first term
a + 3d = fourth term
a + 7d = eighth term
(a + 3d)² = a(a + 7d)
a² + 6ad + 9d² = a² + ad
9d² = ad
9d = a
Solving for common ratio:
𝑎+3𝑑 9𝑑+3𝑑 12𝑑 𝟒
R= = = =
𝑎 9𝑑 9𝑑 𝟑

67. Three men A, B, and C can do a piece of work in t hours working together. Working alone,
A can do the work in 6 hours more, B in 1 hour more, and C in twice the time all working
together. How long would it take to finish the work if all working together?
Solution:
Let t = number of hours A, B & C
together can do the work
t + 6 = number of hours A can do the
work alone
t + 1 = number of hours B can do the
work alone
2t = number of hours C can do the work
Alone
Sum of individual rate = combined work
1 1 1 1
+ + =
𝑡 + 6 𝑡 + 1 2𝑡 𝑡

3t² + 7t – 6 =0
(t + 3) (3t – 2) = 0
2
t = -3 t = x 60 min = 40 min.
3

68. Solve the z if the equation is 4 x 10⁻⁵ = z.


Solution:
4 x 10⁻⁵ = 0.00004

69. Two balls are drawn one at a time from a basket containing 4 black balls and 5 white balls.
If the first ball is returned before the second ball is drawn, find the probability that both
balls are black.

JAY T. OLIVEROS, REE, RME


Engineering Mathematics

Solution:
4 4 16
P= x = = 𝟎. 𝟏𝟗𝟖
9 9 81

70. There are 15 balls in a box: 8 balls are green, 4 are blue and 3 are white. Then 1 green
and 1 blue ball are taken from the box and put away. What is the probability that a blue
ball is selected at random from the box?
Solution:
Green balls = 8 – 1 = 7
Blue balls = 4 – 1 = 3
White balls = 3
Total number of balls in a box = 13
𝟑
Probability that blue ball is selected =
𝟏𝟑

71. Which of the following is equivalent to (x)(x)(x)(x³), for all x?


Solution:
(x) (x) (x) (x³) = (x³) (x³)
= x⁶

72. A number between 1 and 10000 is randomly selected. What is the probability that it will
be divisible by 4 and 5?
Solution:
(4) (5) = 20
1
P = = 𝟎. 𝟎𝟓
20

73. What time after 2 o’clock will the hands of the clock extend in opposite direction for the
first time?
Solution:
𝑥
x = 40 +
12
x = 43.64
direction for the first time = 2:43.64

74. What is the sum of the geometric progression if there are 4 geometric means between 3
and 729?
Solution:
a₁ = 3 n = 6 an = a₆ a₆ = 729
an = a₁𝑟 𝑛−1
a₆ = 3𝑟 6−1
729 =3𝑟 5
r=3
3, 9, 27, 81, 243, 729
Sum of all numbers = 1092

75. A boy on his bicycle to arrive at certain time to a market that is 30 km from his school.
After riding 10 km, he rested for half an hour, and as a result he was obliged to ride the
rest of the trip 2km/hr faster, what was his original speed?
Solution:
Let x = original speed of the boy

JAY T. OLIVEROS, REE, RME


Engineering Mathematics

t₁ = planned time of travel if no delay


30
t₁ = eq. 1
𝑥
t₂ = actual time of travel due to delay
10 20 1
t₂ = + + eq. 2
𝑥 𝑥+2 2
Equate 1 & 2
30 10 20 1
= + +
𝑥 𝑥 𝑥+2 2
40 1
=
𝑥 (𝑥 + 2) 2
x² + 2x -80 = 0
(x + 10) (x – 8) = 0
X = -10x = 8 km/hr.

76. Find the equation whose roots are two times the roots of the equation x³ - 6x² +11x – 6 =
0.
Solution:
To obtain an equation each of whose roots is double to those of eq. x³ - 6x² +11x – 6 = 0,
multiply the 2nd term of the original eq. by 2, the 3rd term by 2² and the 4th term by 2³
= x³ - 6(2)x² + 11(2)²x – 6(2)³ = 0
= x³ - 12x² + 44x – 48 = 0

77. How many 4-digits even numbers can be formed from the digits 0, 1, 2, 3, 4, 5, 6, 7, 8,
and 9 if each digits is to be used only once in each number?
Solution:
₁₀𝑃₄ (10)(9)(8)(7)
= = 𝟐𝟓𝟐𝟎
2 2

78. Rukia has nickels, dimes, and quarters amounting $1.85. If he has twice as many dimes
as quarters, and the number of nickels is two less than twice the number of dimes, how
many quarters does he have?
Solution:
0.05n + 0.1d + 0.25q = 1.85
d = 2q
n = -2 + 2d

n + 2d + 5q = 37
d = 2q
n = 2d -2

n + 2d + 5q = 37
n = 2q – 2

(2q – 2) + 2d + 5q = 37
2q – 2 + (2*2q) + 5q = 37
q=3

79. A club has 25 members, 4 of whom are ECE’s. In how many ways can a committee of 3
be formed so as to include at least one ECE?
Solution:
25 = ₂₅C₃

JAY T. OLIVEROS, REE, RME


Engineering Mathematics

₂₅₋₄C₃ = ₂₁C₃
(35)(24)(23) (21)(20)(19)
₂₅C₃ - ₂₁C₃ = −
3! 3!
= 970

80. If (x-3) is a factor of a polynomial x⁴ - 4x³ - 7x² +kx + 24, what is the value of k?
Solution:
f(x) = x⁴ - 4x³ - 7x² +kx + 24
f(3) = (3⁴) – 4(3)³ - 7(3)² + k(3) + (24) = 0
81 – 108 – 63 + 3k + 24 = 0
3k = 66
k = 22

81. A guy has 8 flowers of different variety. In how many ways can he select 2 or more flowers
to form a bouquet?
Solution:
₈C₂ + ₈C₃ + ₈C₄ + ₈C₅ + ₈c₆ + ₈C₇ + ₈C₈
8∗7 8∗7∗6 8∗7∗6∗5 8∗7∗6∗5∗4 8∗7∗6∗5∗4∗3 8∗7∗6∗5∗4∗3∗2 8∗7∗6∗5∗4∗3∗2∗1
= + + + + + +
2! 3! 4! 5! 6! 7! 8!
= 28 + 56 +70 + 56 + 28 +8 + 1
= 247 ways

82. At a conference, after everyone had shaken hands with everyone else, it was found that
45 handshakes where exchange. How many were at the conference?
Solution:
𝑛(𝑛−1)
Total # of handshake =
2
𝑛 (𝑛 − 1)
= 45
2
n (n – 1) = 90
n² - n – 90 = 0
(n – 10) (n + 9) = 0
n = 10 n = -9
ans.: 10 persons

83. A bag contains 4 white balls and 3 black balls. Another bag contains 3 white balls and
black balls. If one ball is drawn from each bag, determine the probability that the balls
drawn will be 1 white and 1 black.
Solution:
4 5 3 3 𝟐𝟗
𝑥 + 𝑥 =
7 8 7 8 𝟓𝟔

84. If the sides of the right triangle are in A.P., then what is the ratio of its sides?
Solution:
Let (a – s), a, (a + s) be the sides
(a + s)² = a² + (a – s)²
a = 4s
a – s = 4s – s = 3s
a + s = 4s + s = 5s
ratio of its sides = 3:4:5

JAY T. OLIVEROS, REE, RME


Engineering Mathematics

85. If x: y: z = 4: -3: 2 and 2x + 4y - 3z = 20, find x, y, z.


Solution:
Let x = 4k y = -3k z = 2k
2x + 4y - 3z = 20
2(4k) + 4(-3k) – 3(2k) = 20
8k – 12k – 6k = 20
-10k = 20
k = -2
x = 4k = 4(-2) = -8
y = -3k = -3(-2) = 6
z = 2k = 2(-2) = -4

86. How many numbers between 3000 and 5000 can be formed from the digits 0, 1, 2, 3, 4,
5, 6 if repetition is not allowed?
Solution:
Numbers between 3000 and 5000
Thousands place= 2 possible case (3 and 4)
Hundreds place = 6 possibilities (0,1,2,5,6
and 3 0r 4)
Tens place = 5 possibilities (the remaining
digits)
Ones place = 4 possibilities
= 2*6*5*4 = 240

87. Find the mean proportional between


88. How many liters of a 25% acid solution must be added to 80 liters of a 40% acid solution
to have a solution that is 30% acid?
Solution:
25x + 40(80) = 30x + 30(80)
25x + 3200 = 30x + 2400
-5x = -800
x = 160 L

89. A yacht can travel 10 miles downstream in the same amount of time as it goes 6 miles
upstream. If the velocity of the river current is 3 MPH, find the speed of the yacht in still
water.
Solution:
Let s = speed of the boat in still water
S – 3 = speed of the boat downstream
S + 3 = speed of the boat upstream
10 6
=
𝑠+3 𝑠−3
10s – 30 = 6s + 18
10s – 6s = 18 + 30
4s = 48
S = 12 MPH

90. Determine the 5th term of the sequence whose sum of n terms is given 2𝑛+3 – 5.

Solution:

JAY T. OLIVEROS, REE, RME


Engineering Mathematics

A₅ = 5th term
A₅ = S₅ - S₄
A₅ = (25+3 − 5)-(24+3 − 5)
A₅ = 251 – 123
A₅ = 128

91. Find the sum of the first five terms of the geometric progression if the third term is 144 and
the sixth term is 486.
Solution:
an = a₁𝑟 𝑛−1
a₆ =a₃𝑟 6−3
486 = 144𝑟 3
r = 1.5
486 = a₁ (1.5)6−1 ,
a₁ = 64

64, 96, 144, 216, 324, 486


Sum of the first five terms = 64 + 96 + 144 + 216 + 324 = 844

92. A and B working together can finish a job in 5 days, B and C together can finish a job in 4
days, and A and C in 2.5 days. In how many days can all of them do the job working
together?
Solution:
Let a, b, c = No. of days required by
each working done to do the job
T = no. of days to finish the job if all
working together
1 1 1
, , = part of the job done by a, b, c
𝑎 𝑏 𝑐
in one day
1 1 1 1 1 1 1 1 1
( + )+( + )+( + )= + +
𝑎 𝑏 𝑏 𝑐 𝑎 𝑐 4 3 2.5
2 2 2
+ + = 0.983
𝑎 𝑏 𝑐
1 1 1 1
+ + = 0.492 =
𝑎 𝑏 𝑐 𝑇
T = 2.03 days

93. If Chicago is 10% taller than Ishida and Ishida is 10% taller than Chad, then Chicago is
taller than Chad by how many percent?
Solution:
Let x = height of Chad
1.1x = height of Ishida
1.1(1.1x) = height of Ichigo
1.1(1.1x)-x = 0.21 (100%)
= 21%

94. After the price of petroleum oil went up by 10%, a buyer reduced his oil consumption by
the same percent. By what percent would his petroleum bill changed?
Solution:
Let B = original bill

JAY T. OLIVEROS, REE, RME


Engineering Mathematics

x= original price
y = original consumption
B = xy = original bill
The new bill = (1.1) (.9y) = .99xy
B – NB = xy – 0.99xy = 0.01 or 1%

95. Find the mean, median, and mode respectively of the following numbers: 13, 13, 14, 12,
11, 10, 9, 11, 8, 11, 5, and 15.
Solution:
∑𝑥 132
Mean = = = 𝟏𝟏
𝑁 12
22
Median = = 𝟏𝟏
2
Mode = most occurrence of one
number which is 11

96. There are 4 white balls and 6 red balls in a sack. If the ball are taken our successively (the
first ball is not replaced), what is the probability that the balls drawn are of different colors.
Solution:
4 6 6 4 𝟖
Pr = 𝑥 + 𝑥 =
9 10 9 10 𝟏𝟓

97. Solve for x in the following equation: x + 3x + 5x + 7x + … + 49x = 625


Solution:
d = a₂ - a₁
d=3–1
d=2
an = a₁ + (n – 1)d
an = 1 + (n – 1)2
49 = 2n – 1
50 = 2n
n = 25
𝑛
Sn = (𝑎1 + an )
2
25
S₂₅ = (1 + 49)
2
S₂₅ = 12.5 (50)
S₂₅ = 625
625x = 625
X=1

98. An organization consists of n engineers and n nurses. If two of the engineers are replaced
by two other nurses, then 51% of the group members will be nurses. Find the value of n.
Solutions:
Let n – 2 = # of engineers
n + 2 = # of nurses
𝑛+2
( )100 = 51
2𝑛
100n + 200 = 102n
200 = 102n – 100n
200 = 2n
n = 100

JAY T. OLIVEROS, REE, RME


Engineering Mathematics

99. In a certain family, the sum of the parent’s age is twice the sum of their children’s age.
Five years ago, the sum of the parent’s age was four times the sum of the children’s ages
during that time. In fifteen years, the sum of the parent’s age will be equal to the sum of
their children’s ages. How many children are there in the family?
Solution:
Let x = sum of children’s age now
2x = sum of parent’s age now
n = # of children in the family
past present future
parents 2x – 10 2x 2x + 30
children x – 5n x x + 15n

(2x – 10) = 4(x -5n) eq. 1


Future age:
(2x + 30) = (x + 15n) eq. 2
Equate 1 & 2
(X – 15n = -30) multiply all terms by 2
2x - 30n =- 60
- 2x +20n =10
- 10n = -50
n=5
100. Z varies directly as x and inversely as
y². If x = 1 and y = 2, then z =2. Find z
when x = 3 and y = 4.
Solution:
𝑘𝑥
Z=
𝑦2
𝑘 (1)
2=
(2)²
k=8
value of z when x = 3 and y = 4
(8)(3)
z= = 1.5
(4)²

101. When two dice are thrown, what is


the probability that the sum of two
faces shown is 6?
Solution:
The number of possible outcomes is 36 and there are 5 ways to get a sum of 6. (1, 5), (2,
4), (3, 3), (4, 2), (5, 1).
𝟓
So the answer is
𝟑𝟔

102. An ECE class of 40 students took


examination in Electronics and Communications. If 30 passed in Electronics, 36 passed
in Communication and 2 failed in both subjects, how many students passed in both
subjects?
Solution:

JAY T. OLIVEROS, REE, RME


Engineering Mathematics

Let x = no of students who passed in


both subjects
(30 – x) + (36 – x) + x = 38
66 – 38 = x
28 = x

103. The excess of the sum of the fourth and


fifth parts over the difference of the half and third parts of a number is 119. Find the
number.
Solution:
Let x be the number
𝑥
= half part of x
2
𝑥
= third part of x
3
𝑥
= fourth part of x
4
𝑥
= fifth part of x
5

s₁ = sum of 4th and 5th parts


𝑥 𝑥
S₁ = +
4 5
S₂ = difference of half and 3rd parts
𝑥 𝑥
S₂ = −
2 3
The excess of S₁ over S₂ = 119
𝑥 𝑥 𝑥 𝑥
( + ) − ( − ) = 119
4 5 2 3
X = 420

104. What is the area in square feet, of the


triangle whose sides have lengths equal to 10, 6 and 8 feet?
Solution:
A = ½ bh
A = ½ (8) (6)
A = 24

105. Solve for x if the equation is 3102 + 9 *


3100 + 3100 /3 = x.
Solution:
3100
3102 + 9 * 3100 + =x
3

x = 1.91519306 x 10⁴⁹

= 3′⁰³
106. Of the 80 students in class, 25 are
studying German, 15 French and 13
Spanish. 3 are studying German and French; 4 are studying French and Spanish; 2 are
studying German and Spanish; and none is studying all 3 languages at the same time.
How many students are not studying any of the three languages?

JAY T. OLIVEROS, REE, RME


Engineering Mathematics

Solution:
80 = students in class
German = 25 – (3 + 2) = 20
Spanish = 13 – (4 + 2) = 7
French = 15 – (3 + 4) = 8
German & French = 3
German & Spanish = 2
French & Spanish = 4

80 – (20 + 3 + 7 + 2 + 8 + 4) = 36

107. There were 2 small circles C1 and C2


inside a large circle AB. AB is a diameter of the large circle. The centers C1 and C2 of
the smaller circles are on AB. The two small circles are congruent and tangent to each
other and to the larger circle. The circumference of circle C1 is 8 pi. What is the area of
the large circle?
Solution:
A

C₁

C₂

B
Circumference of C₁ = 8𝜋
C = 2 𝜋𝑟
8𝜋 = 2𝜋𝑟
r=4
d = 8 =radius of the larger circle
𝐴 = 𝜋𝑟 2
𝐴 = 𝜋82
𝐴 = 𝟔𝟒𝛑

108. DE is parallel to CB and (length of


AE/length o EB) is 4. If the area of
triangle AED is 20 square inches, what
is the area, in square inches of the
triangle ABC?

Solution:
A

JAY T. OLIVEROS, REE, RME


Engineering Mathematics

D E

C B

𝐴𝐸
=4
𝐸𝐵
AE = 4EB
𝐴𝐵 𝐴𝐸+𝐸𝐵 4𝐸𝐵+𝐸𝐵 𝟓
=( )= =
𝐴𝐸 4 𝐸𝐵 4𝐸𝐵 𝟒

109. Round (202)² to the nearest hundred.


Solution:
(202)² = 40804
Round to the nearest hundred = 40800

110. If w workers, working at equal rates,


can produce x toys in n days how many days it takes c workers, working at the same equal
rates, to produce y toys?
Solution:
The rate r as number of toys per worker is given by
𝑥
r=
(𝑛 𝑥 𝑊)
If N is number of days needed to produce y toys by c workers at same rate then;
𝑦 𝑥
r= =
(𝑁∗𝑐) (𝑛 𝑥 𝑤)
Solve for N
(𝒚∗𝒏∗𝒘)
N=
(𝒄∗𝒙)

111. A number of the form 213ab, where a


and b are digits, has a remainder less than 10 when divided by 100. The sum of all the
digits in the above number is equal to 13 find the digit b.
Solution:
213ab may be written as 213ab = 21300 + 10a + b. When 213ab is divided by 100 the
remainder is 10a + b. The only way for the remainder to be less than 10 is that a = 0
2 + 1 + 3 + 0 + b = 13
b = 13 – 2 -1 -3
b=7

112. Find a negative value of x that satisfies


the equation [(x + 1)² - (2x + 1)½ + 2ꞁxꞁ - 6 = 0.
Solution:
[(x + 1)² - (2x + 1)½ + 2ꞁxꞁ - 6 = 0
Expand
[x² + 2x + 1 – (2x + 1)] ½ + 2ꞁxꞁ - 6 = 0
ꞁx²ꞁ½ + 2ꞁxꞁ - 6 = 0
ꞁxꞁ + 2 ꞁxꞁ - 6 = 0
3 ꞁxꞁ = 6

JAY T. OLIVEROS, REE, RME


Engineering Mathematics

ꞁxꞁ = 2
x = -2

113. If thrice the smaller number exceeds


the larger by 12. Find the larger number I the numbers are consecutive odd integers.
Solution:
Let x = integer
2x -1 = smaller number
2x + 1 = larger number
3 (2x -1) = 12 + (2x + 1)
Solve for x
6x – 3 = 12 + 2x + 1
6x – 2x = 12 + 1 + 3
4x = 16
X=4
Larger number = 2x + 1
= 2(4) + 1
=9

114. Determine how much water should be


evaporated from 50kg of 30% solution to produce a 60%salt solution. All percentages by
weight.
Solution:
30% (50) = (60%) (50 – x)
.30(50) = .60 (50 – x)
15 = 30 - .60x
.60x = 30 – 15
.60x = 15
X = 25 kg

115. A runs around a circular track in 60


seconds, and in 50 seconds. Five seconds after A starts, B starts from the same point in
the same direction. When will they be together for the first time, assuming they run around
the track continuously?
Solution:
Solve for t
𝑐 𝑐
(t – 5) ( ) − (𝑡) ( ) = 0
50 60
Expand
𝑐 𝑐 𝑐𝑡 𝑐
(t – 5) ( ) − (𝑡) ( ): −
50 60 300 10
𝑐𝑡 𝑐
− =0
300 10
Multiply both sides by 300
𝑐𝑡 𝑐
. 300 − . 300 = 0. 300
300 10
Simplify
Ct – 30 c = 0

JAY T. OLIVEROS, REE, RME


Engineering Mathematics

Add 30c to both sides


ct – 30c + 30c = 0 + 30c
ct = 30c
t = 30

116. An antelope is now 50 of her leaps


ahead of a cheetah which is pursuing her. How many more leaps will the antelope take
before it is overtaken if she takes 5 leaps while the cheetah lakes 4 leaps, but 2 of the
cheetah’s leaps are equivalent to 3 of the antelope’s leap?
Solution:
Let a = # of leaps of antelope
c = # of leaps of cheetah
𝑎 5
=
𝑐 4
5𝑐
a=
4
2 leaps of cheetah = 3 leaps of antelope = L
𝐿
1 leap of cheetah =
2
𝐿
1 leap of antelope =
3

For cheetah to catch the antelope:


𝐿 𝐿
(a + 50) ( ) = c ( )
3 2

5𝑐 1 1
( + 50) ( ) = 𝑐 ( )
4 3 2

C = 200
117. Line L passes through the points (-2, 0)
and (0, a). Line LL passes through the points (4, 0) and (6, 2). What value of a makes the
two lines parallel?
Solution:
𝑎−0 𝑎
Slope = =
0−(−2) 2

For line LL (4, 0) (6, 2)


2−0
Slope = =1
6−4

For lines to be parallel, slope have to be equal:


𝑎
=1
2
a=2

118. Solve for x if the equation is 10⁴(5⁴ -


2⁴) / 21 = x
Solution:
104 (54 −24 )
X=
21

JAY T. OLIVEROS, REE, RME


Engineering Mathematics

10,000 (625−16)
X=
21
X = 290, 000

119. Two dice are tossed. What is the


probability that the sum of the two dice is greater than 3?
Solution:
There are 3 ways that the sum is less than or equal to 3. (1,1), (1,2), (2, 1) which means
3 1
=
36 12
The ones that are greater than 3 are
1 𝟏𝟏
1- =
12 𝟏𝟐

120. If L is a line through the points (2, 5)


and (4, 6), what is the value of k so that the point of coordinates (7, k) is on the line L?
Solution:
(𝑦 2 −𝑦 1 )
y - y₁ = (𝑥 − 𝑥₁)
(𝑥 2 −𝑥 1 )
(6−5)
y–5= (𝑥 − 2)
(4−2)
1
y – 5 = ( ) (𝑥 − 2)
2
1
k – 5 =( ) (7 − 2)
2
5
k–5=
2
5
k= +5
2
𝟏𝟓
k=
𝟐

121. Find the negative value of k so that the


graph of y = x² - 2x + 7 and the graph of y = kx + 5 are tangent?
Solution:
x² - 2x + 7 = kx + 5
x² - 2x – kx + 2 = 0
x² -x (2 + k) + 2 = 0
if tangent, determinants must be 0
(2 + k)² - 4(2) = 0
k² + 4k + 4 – 8 = 0
k² + 4k -4 = 0
k = -2 -2√𝟐

122. The circle of equation (x – 3)2 + (y –


2)2 = 1 has center O. Point M (4, 2) is on the circle. N is another point on the circle so that
angle MON has a size of 30°. Find the coordinates of point N.

Solution:
n

JAY T. OLIVEROS, REE, RME


Engineering Mathematics

o m

c
vector CN = vector CO + vector ON
o (3, 2); vector OC = < 3, 2>
ON = 1; O = M same y coordinates
Therefore OM = II to x axis
Von = <1, cos (30°), 1.sin (30°)>
√3 1
=< , >
2 2
1 1 √3 5
<3, 2> + < > √3 , > = < 3 + ,
2 2 2 2
√𝟑 𝟓
Coordinates of N = ( 𝟑 + , )
𝟐 𝟐

123. Vectors u and v are given by u = (2, 0)


and v = (-3, 1). What is the length of vector w given by w = -u – 2v?
Solution:
w = -<2, 0> -2 <-3, 1>
= <-2, 0> + <6, -2>
= <4, -2>
The length of w is given by:
= √(4)2 + (−2)²
= 2√𝟓

124. What is the smallest distance between


the point (-2, -2) and a point on the circumference of the circle given by (x -1)² + (y-2)² =
4?
Solution:
(x -1)² + (y-2)² = 4
Center = 1, 2
Radius = 2
Distance between the center of the circle and point (-2, -2)
= √(−2 − 1)2 + (−2 − 2)²
=5
The shortest distance between point (-2, -2) and a point on a circle is
5–2=3

125. What is the equation of the horizontal


asymptote of function: f(x) = 2/(x + 2) – (x + 3) /(x + 4)?
Solution:
2 −(𝑥+3)
If x increases indefinitely, will approach to 0 and will approach -1. If x decreases
𝑥+2 𝑥+4
2 −(𝑥+3)
indefinitely will approach 0 and will approach -1. Therefore, the equation of
𝑥+2 𝑥+4
horizontal asymptote is y = -1.

126. The lines with equations x + 3y = 2 and


-2x + ky = 5 are perpendicular for k = ?

JAY T. OLIVEROS, REE, RME


Engineering Mathematics

Solution:
Find the slope of x + 3y = 2
−1 2
y= x+
3 3
−1
Slope =
3

Find the slope of -2x + ky = 5


ky = 2x + 5
2 5
y= 𝑥+
𝑘 𝑘
2
Slope =
𝑘
For the lines to be perpendicular, the product of their slope must be equal to -1
1 2
(- ) ( ) = −1
3 𝑘
𝟐
k=
𝟑

127. If f(x) = (x-1)² and g(x) =√x, then (g ˳


f)(x) = ?
Solution:
(g₀f) (x) = g (f (x) )
Substitute f (x) by (x -1)²
(g₀f) (x) = g [(x – 1)²]
= √(𝑥 − 1)²
= |x - 1|

128. The domain of f(x) =

√(4 − x²)/ √(x 2 − 1) is given by the interval


Solution:
2 conditions:
1) (x² - 1) > 0
2) (4 - x²) > 0

1) (x – 1) (x + 1) > 0
[-1, 0) U (0, 1]
2) (-2 – x) (2 + x)
[-2, 2]
Therefore;
[-2, -1) U (1, 2]

129. The area of the circle x² + y² -8y -48 = 0


Is
Solution:
x² + y² -8y -48 = 0
y² - 8y = y² - 8y + 16 -16
= (y – 4)² - 16
x² + (y – 4)² - 16 -48 = 0

JAY T. OLIVEROS, REE, RME


Engineering Mathematics

x² + (y – 4)² = 64
radius = 8
𝐴 = 𝜋𝑟 2
𝐴 = 𝜋(8)2
𝐴 = 𝟔𝟒𝝅

130. An investor has P100, 000, part of


which he invested at 12% interest and the rest at 18%. He received a total annual interest
of P15, 300. How much did he invest at 18% interest rate?
Solution:
Let x = amount of invested at 12%
y = amount of invested at 18%
100,000 = total amount of money
invested in both account
150,300 = total interest earned in both
Account
[x+y=100,000]12= 12x + 12y = 1200000
[.12x + .18y = 15,300] multiply both
sides with 100 to clear the decimals

12x + 18y = 1530000


-12x + 12y = 1200000
6y = 330000
Y = 55,000

131. For what value of k will the two


equations 2x + 4 = 4(X – 2) and –x + k = 2x – 1 have the same solution?
Solution:
2x + 4 = 4(X – 2)
2x + 4 = 4x – 8
-2x = -12
X=6
Substitute 6 to the second equation
–x + k = 2x – 1
-6 + k = 2 (6) – 1
k = 12 – 1 + 6
k = 17

132. An object travels at fifteen feet per


minute. How many feet does it travel in 24 minutes and 40 seconds?

Solution:
74
24’40’’ =
3
𝑓𝑡 74
15 𝑥 𝑚𝑖𝑛 = 370 ft.
𝑚𝑖𝑛 3

133. Solve for x if the equation is 4 /

JAY T. OLIVEROS, REE, RME


Engineering Mathematics

(√20 − √12 ) = 𝑥
Solution:
4
= √𝟓 + √𝟑
√20− √12

134. DE is parallel to CB and (length of


AE/length of EB) is 4. If the area of triangle AED is 20 square inches, what is the area in
square inches, of triangle ABC?

Solution:
A

D E

C B

𝐴𝐸
=4
𝐸𝐵
AE = 4EB
𝐴𝐵 𝐴𝐸+𝐸𝐵 4𝐸𝐵+𝐸𝐵 𝟓
=( )= =
𝐴𝐸 4 𝐸𝐵 4𝐸𝐵 𝟒

135. If a and b are both even numbers,


which of the following could be and odd integers?
𝒂+𝟏
Ans.:
𝒃+𝟏

136. If in is a positive integer such that


n!/(n-2)! = 342, find n.
Solution:
Expand n!
n! = n (n – 1) (n -2)!
𝑛! 𝑛 (𝑛−1)(𝑛−2)!
(𝑛−2)!
= (𝑛−2)!
= n (n – 1)
n (n – 1) = 342
n = 19

137. What is the sum of the reciprocals of


the solutions to the equation: x² - (3/5)x = -11/3.
Solution:
Write the given equation in standard form
3 11
x² - 𝑥 + =0
5 3
𝑏 3 𝑐 11
- = =
𝑎 5 𝑎 3

3⁄ 𝟗
S = 11 5 =
⁄3 𝟓𝟓

JAY T. OLIVEROS, REE, RME


Engineering Mathematics

138. A number is given as 987562153ab


where a and b are digits. Which values of a and b, such that a + b = 11 and a < b, would
result in 987562153ab being divisible by 4?
Solution:
10a + b = 4k
a + b = 11 or b = 11 – a
Substitute b
10a + 11 – a = 4k
4𝑘−11
a=
9
For k = 14
(4 𝑥14−11)
a= =5
9
b = 11-a
b = 11-5 = 6

139. AC is parallel to DE. AE, FG and CD


intersects at the point B. FG is perpendicular to AC and DE. The length of DE is 5 inches,
the length of BG is 8 inches and the length of AC is 6 inches. What is the area in square
inches, of triangle ABC?
Solution:
A
D
B
F G

E
C
𝐷𝐸 𝐵𝐺
=
𝐴𝐶 𝐹𝐵
Solve for FB
5 8
=
6 𝐹𝐵
48
FB =
5

1
A= 𝑏ℎ
2
48
A = ½ ( ) (6)
5
A = 28.8 in²

140. Points A, B, and C are defined by their


coordinates in a standard rectangular system of axes. What positive value of b makes
triangle ABC a right triangle with AC its hypotenuse?
Solution:

B(4,b)

JAY T. OLIVEROS, REE, RME


Engineering Mathematics

A(1,1) C (6,1)

Use Pythagora’s theorem to write that AB² + BC² = AC²


Use coordinates of points to rewrite above as
(4-1)² + (b-1)² + (6-4)² + (1-b)² = (6-1)² + (1-1)²
Simplify as
13 + 2(b-1)² = 25
b = 1 +√𝟔

141. A vendor goes to market to buy fruits


for resale at his store. He spends half of his money for mangoes, and one third of what
remains for bananas. He spends 150 for other fruits and he has 200 left from the amount
he originally had. How much money did he have at the start?
Solution:
Let x = original money
𝑥
= 𝑚𝑎𝑛𝑔𝑜
2
1
𝑥 = 𝑏𝑎𝑛𝑎𝑛𝑎𝑠
3
X – 150 = other fruits

𝑥 1 𝑥
+ ( ) + 150 = 𝑥 − 200
2 3 2
𝑥 𝑥
x - − ( ) = 150 + 200
2 6
6𝑥 − 3𝑥 − 𝑥
= 350
6
2𝑥
= 350
6
X = 1050

142. Seven carpenters and 5 masons earn a


total of 2, 300 per day. A t the same rate of pay 3 carpenters and 8 masons earn 2, 040.
What are the wages per day of the carpenter and a mason?
Solution:
x = carpenter rate
y = mason rate
[3x + 8y = 2040]7
[7x + 5y = 2300]3
Subtract
21x + 56y = 14280
21x + 15 y = 6900
41y = 7380
y = 180

JAY T. OLIVEROS, REE, RME


Engineering Mathematics

3x + 8y = 2040
3x + 8 (180) = 2040
3x = 2040 – 1440
X = 200
Ans.:(180, 200)

143. A man and a boy can do 15 days a


piece of work which would be done by 7 men and 9 boys in 2 days. How long would it
take one man do it alone?
Solution:
Let x = man-day
y = boy-day
complete job = 1
A man and a boy can do in 15 days of work
15x + 15y = 1
7 men and 9 boys done the work in 2 days
14x + 18y = 1
Equate
[15x + 15y = 1]6
[14x + 18y = 1]5
90x + 90y = 6
70x + 90y = 5
20x = 1
20 man-day are required to do the job therefore it would take 20 days for 1 man do the job
alone.

144. A certain two-digit number is 1 less


than five times the sum of its digits. If 9 were added to the number, its digits would be
reversed. Find the number?
Solution:
10x + y = 5(x +y) – 1
10x + y = 5x + 5y – 1
5x – 4y + 1 = 0
If 9 were added to the number, its digit would reverse
(5x – 4y) +9 = 1
5x – 4y = -8 = 4x – 5y = -8
[4x – 5y = -8]5
[5x – 4y = -1]4
20x – 25y = -40
20x – 16y = -4
-9y = -36
y=4
20x – 25y = -40
20x – 25(4) = -40
20x = -40 + 100
20x = 60
X=3

JAY T. OLIVEROS, REE, RME


Engineering Mathematics

xy=34

145. If one root of 9x² - 6x + k = 0 exceed


the other by 2, find the value of k.
Solution:
9x² - 6x + k = 0
a = 9 b = -6 c = k
−𝑏
(r₂ + 2) r₂ =
𝑎
−6
2r₂ = (−2)
9
−6−18 −24 −4
r₂ = = =
2(9) 18 3

𝑐
r₂² (2r₂) =
𝑎
𝑘
r₂² (2r₂) =
𝑎
4 4 𝑘
(- )² + 2(− ) =
3 3 9
16 6
k = 9[ − ]
9 3
k = -8

146. Solve: √2x − 5 − √x − 2 = 2.


Solution:
√2x − 5 − √x − 2 = 2
X = 27

147. A spee d boat going across a lake 8 km


wide proceeds 2 km at a certain speed and then completes the trip at a speed ½ km/hour
faster. By doing this, the speed arrives 10 minutes earlier than if the original speed had
been maintained. Find the original speed of the speed boat.
Solution:
Let s = the original speed
(s + .5) = the faster speed for last 6 km of the trip
1
10 min = ℎ𝑟
6

Find the time it took at the 2 speed


2 6 2. (𝑠 + .5) + 6𝑠
+ =
𝑠 𝑠 + .5 5 (𝑠 + .5)
2𝑠+1+6𝑠 8𝑠 + 1
= =
𝑠 (𝑠+ .5) 𝑠 (𝑠+ .5)

Find s when the difference is 10 min


8 8𝑠 + 1 1
− =
𝑠 𝑠 (𝑠 + .5) 6
S = 4 km/hr

JAY T. OLIVEROS, REE, RME


Engineering Mathematics

148. An audience of 540 people is seated in


rows having the same number of persons in each row. If 3 more persons seat in each row,
it would require 2 rows less to seat the audience. How many persons were in each row
originally?
Solution:
Let r = rows
p = number of persons
rp = (r – 2) (p + 3)
rp = rp – 2p + 3r – 6
3𝑟−6
p=
2

540 = rp
3𝑟−6
540 = r ( )
2
1080 = 3r² - 6r
360 = r² - 2r
r² - 2r – 360 = 0
(r- 20) (r + 18)
r = 20 r = -18

If r = 20 then
3𝑟−6 3 (20)−6
p= = = 𝟐𝟕
2 2

149. Find the third proportional to 4 and 12.


Solution:
4:12 = 12:p
12 𝑥 12
p= = 𝟑𝟔
4

150. How many terms of the progression 4,


7, 10, 13,… must be taken so that the sum will be 69?
Solution
a = 4 d = 3 Sn = 69

𝑛
Sn = [2𝑎 + (𝑛 − 1)𝑑 ]
2
𝑛
69 = [2(4) + (𝑛 − 1)3]
2
𝑛
69 = [8 + 3𝑛 − 3]
2
n=6

151. Determine x so that 2x + 1, x² + x + 1,


3x² -3x +3 are consecutive terms of arithmetic progression.
Solution:
2nd term – 1st term = 3rd term – 2nd term
(x²+x+1)-(2x+1) = (3x²-3x+3)-(x²+x+1)
x² - x = 2x² - 4x + 2

JAY T. OLIVEROS, REE, RME


Engineering Mathematics

-2x² + x² + 4x – x -2 = 0
[-x² + 3x – 2 = 0]-1
x² - 3x + 2 = 0
(x – 2) (x – 1) = 0
x=2 x=1

152. An equipment cost P50, 000.00 and


depreciates 20% of the original cost during the first year, 16% during the second year,
12% during the third year, and so on, for 5 years. What is the value at the end of 5 years?
Solution:
Year Net Book Depreciation
Value
1 50,000 x 20% 10,000
2 40,000 x 16% 6,400
3 33,600 x 12% 4,032
4 29,568 x 8 % 2,365.44
5 27,202.56 x 4% 1,088.10

Value at the end of 5 years = 27,202.56 - 1,0 88.12


= 26,114.46

153. Find the sum of the first 100 positive


integers that is exactly divisible by 7.
Solution:
n = 100 a₁ = 7 d = 7
𝑛
Sn = [2𝑎 + (𝑛 − 1)𝑑 ]
2
100
Sn = [2(7) + (100 – 1)7]
2
Sn = 35, 350

154. Find the 50th term of a geometric


progression if the 20th term is 1200 and the 30th term is also 1200.
Solution:
an = a₁𝑟 𝑛−1
a₃₀ = 𝑎₂₀𝑟 30 −20
1200 = 1200𝑟10
r=1
a₅₀ = 1200(1)50−30
a₅₀ = 1200

155. A woman started a chain letter by


writing to four friends and requesting each to copy the letter and sent it to four other friends.
Of the chain was unbroken until the 5th set of letters was mailed, how much was spent for
postage at P8.00 per letter?
Solution:
(4)ᶦ + (4)² + (4)³ + (4)⁴ +(4)5

JAY T. OLIVEROS, REE, RME


Engineering Mathematics

4+16+64+256+1024 = 1364 (8) = 10,912

156. A soccer ball is dropped from height of


6 meters. On each rebound it rises 2/3 of the height from which it last fell. What distance
has it traveled the instant it strikes the ground for the 7th time?
Solution:
1st term a = initial height = 6
2
2nd term = 1st rebound = (6) ( ) = 4
3
Total distance travelled at 7th time in upward level and downward level
1st term= 6
2
Common ration =
3
𝑎(1−𝑟 7 )
Sum of 7 terms =
1−𝑟
2
6(1−( )7 )
3
= 2
1−
3
4118
= (downward travel)
243

For upward travel


1st term = 4
2
Common ration =
3
2
4(1−( )6 )
3
Sum of 6 terms = 2
1−
3
2660
=
243
4118 2660
Total travel = + = 𝟐𝟕. 𝟖𝟗 𝒎
243 243

157. The arithmetic mean of two numbers


is 4, and their harmonic mean is 15/4. Find the numbers.
Solution:
𝑥+𝑦
Am = =4
2
x+y=8
2𝑥𝑦 15
Hm = =
𝑥+𝑦 4
So,
2𝑥𝑦 15
=
8 4
2𝑥𝑦 30
=
8 8
2xy = 30
x+y=8
x=8–y
2(8 – y) y = 30
8y - y² = 15
y² - 8y + 15 = 0
(y – 3) (y – 5) = 0
Y=3 y=5

JAY T. OLIVEROS, REE, RME


Engineering Mathematics

x+y=8 x+y=8
x+3=8 x+5=8
x=5 x=3

158. Find the real values of x and y


satisfying the given equation: (2x + 3y) + i (3x –5 y) = 8 – i7.
Solution:
(2x + 3y) + i (3x –5 y) = 8 – i7
[2x + 3y = 8]3
[3x – 5y = -7]2
Equate
6x + 9y = 24
6x – 10y = -14
19y = 38
y=2
2x + 3y = 8
2x + 3(2) = 8
2x = 2
x=1

159. From the equation 12x³ - 8x² + kx + 18


= 0, find the value of k if one root is negative of the other.
Solution:
12x³ - 8x² + kx + 18 = 0
One root is a and the other root is –a, 3rd root being p.
12(x-a)(x+a)(x-p) = 12(x²-a²)(x-p)
= 12x³ - 12x²p – 12a²p
-12p = -8
2
p=
3
12a²p = 18
9
a²=
4
3
a=
2
k = -12a²
3
k = -12( )² = -27
2

160. In how many ways can a group of 6


people be seated on a row of 6 seats if a certain 2 refuse to sit next to each other?
Solution:
Let a and b the one who refuse to sit next to each other

There are 2 x 5! Ways of arranging the people so that a and b are next to each other, so
6! – (2x5!) = 480 ways of arranging the people so that a and b are not next to each other

161. How many different 8-digit numbers


can be formed from the digits 2, 2, 2, 5, 5, 7, 7, 7?

JAY T. OLIVEROS, REE, RME


Engineering Mathematics

Solution:
5’s = 2
2’s = 3
7’s = 3

8!
= 𝟓𝟔𝟎
(2!)(3!)(3!)

162. In how many ways can 10 different


magazines be divide among A, B and C so that A gets 5 magazines, B 3 magazines and
C 2 magazines?
Solution:
10!
= 𝟐𝟓𝟐𝟎
(5!)(3!)(2!)

163. What is the probability of drawing 6


white balls from a jar containing 9 white, 4 red, and 3 blue balls?
Solution:
(6!)(9!)
= 6.94 𝑥 10−3
(6!)(9!)(4!)(3!)
= 0.006
= 0.01

164. Ten books consisting of 5 Mathematics


books, 3 physics books and 2 chemistry books are place in a bookcase at random. What
is the probability that the same books are all together?
Solution:
3 blocks of Math, Physics and Chemistry can be arranged in 3! Ways
Within the blocks, book can be arranged in (5!)(3!)(2!) ways
Favorable ways = (3!) (5!) (3!) (2!) = A
Total ways = 10! = B
𝐴 (3!) (5!) (3!) (2!) 𝟏
P= = =
𝐵 10! 𝟒𝟐𝟎

165. In a racing contest, there are 240


vehicles which will have provisions that will last for 15 hours. Assuming constant hourly
consumption for each vehicle, how long will the fuels provisions last if 8 vehicles withdraw
from the race every hour after the first?
Solution:
240(15) = 240(1)+232(1)+224(1)…
= 240n – 8(1+…n-1)
= 0.5 (n-1)n
240(n-15) = 8(0.5(n-1))
15(n-15) = n(n-1)
n² - 16n + 225 = 0
(n-9) (n-25)
n = 9 n=25 (Answer)
ignoring the value less than 15

JAY T. OLIVEROS, REE, RME


Engineering Mathematics

166. A clerk submitted the following


reports. The average rate of production of radios is 1.5 units for every 1.5 hrs. work by 1.5
workers. How many radios will produce in one month by 30 men working 200 hrs. during
the month?
Solution:
𝐻₁ 𝐻₂
𝑊 ₁𝑥 = 𝑊 ₂𝑥
𝑅₁ 𝑅₂
1.5 200
(1.5) ( ) = (30)( )
1.5 𝑅2
R₂ = 4000

167. A piece of rod length 52 cm. is cut into


two unequal parts. Each part is then bent to form a square. It is found that the total area
of the two squares is 97 cm². Find the difference between the sides of each square?
Solution:
Let x = side of 1 square
x² = area of that square
4x = perimeter of that square
Let y = side of the other square
y² = area of the 2nd square
4y = perimeter of the 2nd square

Length of the rod


4x + 4y = 52
X + y = 13
Y = 13 – x
Area of the 2 square
x² + y² = 97
Substitute 13 – x to y
x² + (13 – x)² = 97
x² + (13 – x) (13 – x) = 97
x² + x² - 26 x + 169 – 97 = 0
2x² - 26x + 72 = 0
x² - 13x + 36 = 0
(x – 4) (x – 9) = 0
x=4 x=9
The difference between the sides of each square = (9 – 4) = 5

168. Solve the trigonometric equation:


3 sec²x – 4 = 0.
Solution:
3 sec²x – 4 = 0.
3 𝑠𝑒𝑐²𝑥 4
=
3 3
2
sec²x =
√3
1 2 √3
= = cos x =
cos 𝑥 √3 2

JAY T. OLIVEROS, REE, RME


Engineering Mathematics

𝝅 𝟏𝟏𝝅
X= + 𝝅𝒏, + 𝝅𝒏
𝟔 𝟔

169. In what quadrant will the angle Ө


terminate, if sin Ө is positive and sec Ө is negative?

Sin(+)
Csc (+) All (+)

Tan(+) Cos(+)
Cot(+) sec(+)

Note: in the 2nd quadrant sine is (+) and sec is (-)

170. If sec (2x – 3) = 1/sin(5x – 9 ),


determine the value of x in degrees.
Solution:
1
Sec (2x – 3) =
sin(5𝑥−9)
Sec (2x – 3) = csc (5x – 9)
Sec (2x – 3) = sec [90 – (5x – 9)]
2x – 3 = 99 – 5x
7x = 102
x = 14.57°

171. What is the maximum value of 3 – 2


cos Ө?
Solution:
3 – 2 cos Ө = 3 – 2 (-1)
=3+2
=5

172. Solve the trigonometric equation:


2cosx + 1 = 0
Solution:
2cosx + 1 = 0
2 cos 𝑥 1
= −
2 2
1
Cos x = −
2
1
x=−
2
𝟐𝛑 𝟒𝛑
= + 𝟐𝐧𝛑, + 𝟐𝐧𝛑
𝟑 𝟑

173. If log x + log5 = log (x + 5) what is the

JAY T. OLIVEROS, REE, RME


Engineering Mathematics

value of x?
Solution:
If log x + log5 = log (x + 5)
Log (5x) = log (x + 5)
5x = x + 5
4x = 5
5
x=
4
x = 1.25

174. If the angles of the triangles are 2x, x +


15, and 2x + 15, find the smallest angle in mills.
Solution:
2x + (x + 15) + (2x + 15) = 180
5x = 150
x = 30
2x = 2 (30) = 60
X + 15 = 30 + 15 = 45 (smallest angle)
2x + 15 = 2 (30) + 15 = 75

6400 𝑚𝑖𝑙𝑠
45° x = 𝟖𝟎𝟎 𝒎𝒊𝒍𝒔
360°

175. If (log₁₀X)² = 3 - log₁₀x². Which of the


following choices can be a value of x?
Solution:
(𝑙𝑜𝑔10 𝑥 )2 = 3 − 𝑙𝑜𝑔10 𝑥 2
(𝑙𝑜𝑔10 𝑥 )2 + 𝑙𝑜𝑔10 𝑥 2 = 3
(𝑙𝑜𝑔10 𝑥 )2 + 2𝑙𝑜𝑔10 𝑥 = 3
log10𝑥 = 𝑢
(u)² + 2u = 3
u² + 2u – 3
(u + 3) (u-1)
u= -3; u = 1
𝑙𝑜𝑔10 𝑥 = 1 ∶ 𝑥 = 10
1
𝑙𝑜𝑔10 𝑥 = −3: 𝑥 = = 𝟎. 𝟎𝟎𝟏 𝐨𝐫 𝟏𝟎−𝟑
1000

176. Find the value of x in the equation


2𝑐𝑜𝑠𝑥
√5 = 5.
Solution:
2𝑐𝑜𝑠𝑥
√5 = 5.
x = 0°

177. If 𝑎 𝑥 = 𝑏 𝑦 and 𝑏 𝑝 = 𝑎𝑞 , then


px = qy

JAY T. OLIVEROS, REE, RME


Engineering Mathematics

178. Solve the trigonometric equation:


(3cosx + 7)(-2sinx – 1) = 0.
Solution:
(3cosx + 7)(-2sinx – 1) = 0
3 cosx + 7 = 0 or -2 sinx – 1 = 0
−7 −1
Cos x = or sinx =
3 2
-1< cosx < 1
−1
Sinx = [value of sin x is (-) in 2nd and
2
4th quadrant]
𝜋 7𝜋 1
Sin (𝜋 + ) = sin = −
6 6 2
𝟕𝝅 𝟏𝟏𝝅
X= + 2 𝝅𝒏, + 2 𝝅𝒏
𝟔 𝟔

179. If the bearing of point A from B is S 40°


W, then the bearing of B from A is:
N

E E

Bearing of B from A is N 40° E

180. A clock has a dial face of 12 in. radius.


The minute hand is 9 inches white the hour hand is 6 inches. The plane of rotation of the
hour hand is 2 inches above the plane of rotation of the minute hand. Find the distance
between the tips of the minute and hour hand at 5:40 a.m.
Solution:
At 5:40, minute hand is at 8, which 8x30° = 240°
At 5:00, hour hand is at 5 (5x30°=150° clockwise of 12) and moves at a rate of 30°/hr =
0.05/min
At 5:40, position of hour hand is 150° + 40(0.05) = 170°
Angle between minute hand and hr hand = 240° - 170° = 70°
X = distance between tips of hands (same plane) the hands form a Δ with side of x, with
angle opposite of 70°
x² = 6² + 9² - 2(6)(9) cos (70)
x² = 117-108 cos 70
x² = 80.0618

JAY T. OLIVEROS, REE, RME


Engineering Mathematics

d = distance between the tips of the hands


d² = x² + 2²
d = √80.0618 + (2)²
d = 9.1685

181. Two towers are 60 meters apart from


each other. From the top of the short tower, the angle of elevation of the top of the taller
tower is 40°. How high is the taller tower if the height of the smaller tower is 40m?
Solution:

60 tan 40 = 50
50 m + 40 m = 90m

182. Considering the earth to be a


sphere of radius 6400 km, find the radius of the 60th parallel of latitude.
Solution:
Let r = radius of parallel latitude
re = 6400 km (radius of the earth)
𝑟
cos 60 =
re
r = re cos 60
r = 6400 cos 60
r = 3,200 km

183. Solve the trigonometric equation:


(6tan²x – 2)(2tan²x – 6) = 0.
Solution:
(6tan²x – 2)(2tan²x – 6) = 0
6tan²x-2=0 2tan²x-6=0
2 6
tan²x = tan²x =
6 2
√3
tanx = tanx = √3
3
𝑦
tanƟ =
𝑥
𝝅 𝟓𝝅 𝝅 𝟐𝝅
x= + 𝒏𝝅, +2 𝒏𝝅, + 𝒏𝝅, + 𝒏𝝅
𝟔 𝟔 𝟑 𝟑

184. From a point on a level ground, the


angles of elevation of the top and bottom of the ABS-CBN tower situated on the top of the
hill are measured as 48°and 40°, respectively. Find the height of the hill if the height of the
tower is 116 feet.

Solution:

tan 40 =
𝑥

x= 𝑒𝑞. 1
tan 40

JAY T. OLIVEROS, REE, RME


Engineering Mathematics

ℎ+116
tan 48 =
𝑥
ℎ + 116
x= 𝑒𝑞. 2
tan 48

Equate 1 & 2
ℎ ℎ + 116
=
tan 40 tan 48
htan48 = htan 40 + 116 tan 40
116 tan 40
h=
tan 48−tan 40
h = 358.49 m

185. A ladder, with its foot in the street


makes an angle of 30° with the street when its top rest on a building on one side of the
street and makes an angle of 40° with the street when its top rest on a building on the
other side of the street. If the ladder is 50 ft. long, how wide is the street?
Solution:
𝑥
Cos 30 = 1
50
x₁ = 50 cos 30
x₁ = 43.3
x₂ = 50 cos 40
x₂ = 38.3
d = x₁ + x₂
d = 43.3 + 38.3
d = 81.6 ft.

186. A wall is 15 ft. high and 10 ft. from a


building. Find the length of the shortest ladder which will just touch the top of the wall and
reach a window 20.5 ft. above.
Solution:
By ratio and proportion
15 20.5
=
𝑥 𝑥+19
15x + 150 = 20.5x
5.5x = 150
X = 27.27
L² = (x + 10)² + (20.5)²
L = √(27.27 + 10)2 + (20.5)²
L = 42.54 m

187. A poll tilts toward the sun at an angle


10° from the vertical casts a shadow 9 meters long. If the angle of elevation from the tip
of the shadow to the top of the pole is 43°, how tall is the pole?
Solution:
ℎ 9
=
sin 43 sin 37
9 sin 43
H=
sin 37

JAY T. OLIVEROS, REE, RME


Engineering Mathematics

H = 10.2 m

188. If cos Ө = √3/2, find 1 - tan²Ө..


Solution:
𝑎
Cos Ɵ =

2 y
Ɵ
√3
Using special triangle, y = 1
1
1 - tan² Ɵ = 1 – ( )²
√3
1
=1-
3
𝟐
=
𝟑

189. Solve the trigonometric equation “-


2sec²x + 4 = -2secx “in the interval [0,2Pi].
Solution:
−2𝑠𝑒𝑐 2 𝑥 + 4 −2𝑠𝑒𝑐𝑥
=
2 2
−2𝑠𝑒𝑐 2 𝑥 + 2 = −𝑠𝑒𝑐𝑥
sec²x – secx -2 = 0
(secx – 2) (secx + 1) = 0
Secx = 2 secx = -1
1
Cosx = cos x = -1
2

𝝅 𝟓𝝅
X= , ,𝝅
𝟑 𝟑

190. Solve the trigonometric equation 2sinx


cos(-x) = 2sin(-x)sin(x) in the interval [0, 2Pi].

191. From a helicopter fling at 30, 000 feet,


the angles of depression of two cities are 28° and 55°. How far apart are the two cities?
Solution:
30,000
Cos 62 =
𝐿
L = 30,000cos62
L = 63,901.63
𝑥 𝐿
=
𝑠𝑖𝑛27 sin 125
63901.63 sin 27
X=
sin 125
X = 35, 415.56 ft

192. Two angles are adjacent and form an


angle of 120°. If the larger angle is 20° less than three times the smaller angle, find the
larger angle.

JAY T. OLIVEROS, REE, RME


Engineering Mathematics

Solution:
Let x be the smaller angle
3x – 20= larger angle

X + (3x – 20) = 120


4x = 140
X = 35
3x – 20
3(35) – 20 = 85° larger angle

193. A pine tree broken over by the wind


forms a right triangle with the ground. If the broken part makes an angle of 50° with the
ground and the top of the tree is now 20 ft from its base, how tall was the pine tree?
Solution:
20
Tan 50 =
𝑎
A = 20tan50
A = 23.84
c² = a² + b²
c² = (23.84)² + (20)²
c = √(23.84)2 + (20)²
c = 31.12

h = 23.84 + 31.12
h = 55 ft.

194. A ball, 5 ft in diameter, rolls up an


incline of 18°20’. What is the height of the center of the ball above the base of the incline
when the ball has rolled up 5 ft up the inclined?
Solution:
55
18°20’ =
3
55 𝑎
sin =
3 5
55
a = 5sin = 1.57
3
h = 1.57 + 2.5 = 4ft.

195. If coversed SinӨ = 0.134, find the value


of versed SinӨ.
Solution:
coversed SinӨ = 0.134
coversed sinƟ = 1 – sinƟ
0.134 = 1 – sinƟ
sinƟ = 1 – 0.134
sinƟ = 0.866
Ɵ = 60°
Versed sinƟ = 1 – cosƟ
= 1 – cos 60°

JAY T. OLIVEROS, REE, RME


Engineering Mathematics

= 0.5

196. A vertical pole consists of two parts,


each one half of the whole pole. At a point in the horizontal plane which passes through
the foot of the pole and 36 m from it, the upper half of the pole subtend an angle whose
tangent is 1/3. How high is the pole?
Solution:

m?=x=H

G 36m B

𝐻
mTGB = α tanα =
36
𝐻
2
mMGB = 𝛃 tan𝛃 =
36
mMGT = α – 𝛃

𝑡𝑎𝑛𝛼−𝑡𝑎𝑛𝛽 1
tan(α – 𝛃) = =
1−𝑡𝑎𝑛𝛼 𝑡𝑎𝑛𝛽 3

𝐻 𝐻

36 72
= 𝐻 𝐻
1+( )( )
36 72

𝐻
72
= 𝐻²+2592
2592

1 36 𝐻
=
3 𝐻² + 2592

H²- 108H + 2592 = 0

−𝑏+√𝑏²−4𝑎𝑐
H=X=
2𝑎

−(−108)+ √108²−4(1)(2592)
=
2(1)

H = 72 m

197. Solve the trigonometric equation sin2x


= -sin(-x) in the interval[0, 2pi].
Solution:
sin2x = -sin(-x)

JAY T. OLIVEROS, REE, RME


Engineering Mathematics

sin 2x = sinx
sin2x – sinx = 0
2sinx cosx – sinx = 0
Sinx (2cosx -1) = 0
Sinx = 0 2cosx – 1= 0
2 cos 𝑥 1
=
2 2
1
Cosx =
2
𝝅 𝟓𝝅
X = 𝝅, , 𝝅,
𝟑 𝟑

198. If the sides of the triangle are 2x + 3,


x²+ 3x + 3, and x² + 2x, find the greatest angle.
Solution:
Apply cosine rule
(x+3x+3)²=(2x+3)²+(x²+2x)²-2(2x+3)(x²+2x)cosƟ
(x+3)(2x²+5x+3)-(2x+3)²=-2(2x+3)(x²+2x)cosƟ
(x+3)(2x+3)(x+1)-(2x+3)²=2(2x+3)(x²+2x)cosƟ
(x+3)(x+1)-2x-3=-2(x²+2x)cosƟ
(x²+2x)=-2(x²+2x)cosƟ
1
cosƟ =-
2
Ɵ = 120°

199. ABDE is a square section and BDC is an


equilateral triangle with C outside the square. Compute the value of angle ACE.

A B
15°
60°

30° C
15°
90°

E D

200. The angle of elevation of the top of a


tower from a point A is 23°30’. From another point B, the angle of elevation of the top of
the tower is 55°30’. The point A and B are 217.45 m apart and on the same horizontal
plane as the foot of the tower. The horizontal angle subtended by A and B at the foot of
the tower is 90°. Find the height of the tower.
Solution:
Let x=distance from point A to pole foot
say O
Y=distance from point B to C
x² + y² = 217.45° since the angle made by AOB is 90°
ℎ ℎ
tan(23.5) = tan(55.5) =
𝑥 𝑦

JAY T. OLIVEROS, REE, RME


Engineering Mathematics

𝑥 tan(55.5)
=
𝑦 tan(23.5)
X = √(217.45)² − 𝑦²
√(217.45)2 − 𝑦² tan(55.5)
=
𝑦 tan (23.5)
𝑦 = 62.26
h = ytan (55.5)
= 62.26 tan (55.5)
h = 90.6 m

201. The y coordinates of all the points of


intersection of the parabola y² = x + 2 and the circle x² + y² = 4 are given by
Solution:
x² + x +2 = 4
x² +x -2 = 0
(x + 2) (x -1)
x = -2 x = 1
if x = -2
y² = x + 2
y² = 0
if x = 1
y² = 1 + 2
y² = 3

y = 0,+ √𝟑

202. What is the smallest positive zero of


function f(x) = ½ - sin(3x + Pi/3)
Solution:
1 𝜋
− sin (3𝑥 + ) = 0
2 3
𝜋 1
Sin (3x + ) =
3 2

𝜋 𝜋 𝜋 5𝜋
3x + = + 2𝑘𝜋 and 3x + = + 2𝑘𝜋
3 6 3 6
𝜋 𝜋
3x + = + 2k𝜋
3 6

−𝜋
3x = + 2𝑘𝜋
6
−𝜋 2𝑘𝜋
X= +
18 3
𝜋 5𝜋
3x + = + 2𝑘𝜋
3 6
𝜋
3x = + 2𝑘𝜋
2
𝜋 2𝑘𝜋
X= +
6 3

−𝜋 𝜋
For k = 0, x = and
6 6

JAY T. OLIVEROS, REE, RME


Engineering Mathematics

𝛑
is smallest positive zero
𝟔

203. A cylinder of radius 5 cm is inserted


within a cylinder of radius 10 cm. the two cylinder have the same height of 20 cm. What is
the volume of the region between the two cylinders?
Solution:
Let V₁ = volume of small cylinder
V₂ = volume of large cylinder
V₁ = (5²𝜋) x 20 = 500𝜋
V₂ = (10²𝜋) x 20 = 2000𝜋
V = V₂ - V₁
V = 2000𝜋 − 500𝜋
V = 1500𝝅

204. A data set has a standard deviation


equal to 1. If each data value in the data set is multiplied by 4, then the value of standard
deviation of the new data set is equal to
Answer:
When each value is multiplied by 𝑥1
New standard deviation = Old standard deviation x 𝑥1
So, New standard deviation = 4 x 1 = 4

205. A cone made of cardboard has a


vertical height of 8 cm and has a radius of 6 cm. If this cone is cut along the slanted height
to make a sector, what is the central angle, in degrees, of the sector?
Solution:
P = 2𝜋𝑟
= 2𝜋6
= 12𝜋
H = √𝑟² + ℎ²
= √6² + 8²
= 10
S=Hxt
= 10t
12𝜋 = 10𝑡
180
t = 1.2𝜋 x = 216°
𝜋

206. If sin(x) = -1/3 and Pi ≤ X ≤ 3Pi/2, then


cot(2x) = ?
Solution:
1
Cot (2x) =
tan(2𝑥)
1−𝑡𝑎𝑛²
=
2 tan 𝑥
tan(x) = 𝑠𝑖𝑛𝑥 ⁄𝑐𝑜𝑠𝑥
sin²x + cos²x = 1

JAY T. OLIVEROS, REE, RME


Engineering Mathematics

cos²x = 1 - sin²x
cos x = √1 − 𝑠𝑖𝑛²𝑥 and - √1 − 𝑠𝑖𝑛²𝑥
1
cosx = -√1 − ( )²
3
2√2
=-
3
tan(x) = sin 𝑥⁄cos 𝑥
1
(− )⁄ 2√2
= 3
− 3
1
=
2√2
(1 − 𝑡𝑎𝑛2 )⁄
cot2x = 2 tan 𝑥
1
(1 − )
= 8⁄
1
√2

= 7√2⁄8
𝟕
=
(𝟒√𝟐)

207. If in a triangle ABC, sin(A) = 1/5, cos(B)


= 2/7, then cos(C) = ?
Solution:
A + B + C = 180°
cos (180° - C ) = cos (A + B)
cos (180°)cos(C) + sin(180°)sin(C) =
cos(A)cos(B) – sin(A)sin(B)
cos(C) = sin(A)sin(B) – cos(A)cos(B)
1
sin(A) =
5
2
cos(B) =
7
1 2√6
cos(A) = √1 − ( )² =
5 5
2 √45
sin(B) = √1 − ( )² =
7 7
Substitute the value of sin(A), sin(B),
cos(A) and cos(B) into cos(C) =
sin(A)sin(B) – cos(A)cos(B)
(√𝟒𝟓 − 𝟐√𝟐𝟒)⁄
cos(C) = 𝟑𝟓

208. What value of x makes the three terms


x, x/(x +1) and 3x/[(x + 1)(x + 2)] those of a geometric sequence?
Solution:
Let r be the common ratio
𝑥 3𝑥 𝑟𝑥
= 𝑟𝑥 and =
𝑥+1 [(𝑥+1)(𝑥+2)] 𝑥+1
Solve for x

JAY T. OLIVEROS, REE, RME


Engineering Mathematics

1 3
r= and r =
𝑥+1 (𝑥+2)
1 3
=
(𝑥 + 1) (𝑥 + 2)

3𝑥 + 3 = 𝑥 + 2
2x = -1
𝟏
X=−
𝟐

209. The sum of the sides of the triangle


is equal to 100 cm. if the angles of the triangle are in the continued proportions of 1: 2: 4.
Compute the shortest side of the triangle.
Solution:
A, B, C are 1:2:4
There are 7 parts so angles are
1 2 4
+ + = 180
7 7 7
180 2 𝑥 180 4 𝑥 180
Sin ( ) ∶ sin ( ) :sin ( )
7 7 7
.434: .782:.975
Total = 2.191
A + B + C = 100

.434 𝑥 100
A= = 19.808
2.191

.782 𝑥 100
B= = 35.691
2.191

.975 𝑥 100
C= = 44.5
2.191

19.808 + 35.691 + 44.5 = 100


Shortest side = 19.808

210. The sides of the triangular field which


contains an area of 2400 sq.cm. are in continued proportion of 3: 5: 7. Find the smallest
side of the triangle.
Solution:
Let a, b, c, be the 3 sides
S = (a + b + c)/2
a = 3x b = 5x c = 7x
S = 15x/2
S = 7.5x
A = 2400

2400 = √𝑠(𝑠 − 𝑎)(𝑠 − 𝑏)(𝑠 − 𝑐)


2400 = √7.5𝑥(7.5𝑥 − 3𝑥)(7.5𝑥 − 5𝑥)(7.5𝑥 − 7𝑥)

JAY T. OLIVEROS, REE, RME


Engineering Mathematics

2400 = √42.1875𝑥⁴
Square both sides
5, 760, 000 = 42.1875x⁴
136533.33 = x⁴
X = 19.2225
3x = 3(19.225) = 57.77 smallest side
5x = 5(19.2225) = 96.1125
7x = 7(19.2225) = 134.58

211. In triangle ABC, angle A = 80 deg. And


Point D is inside the triangle. If BD and
CD are bisectors of angle B and C, solve for the angle BCD.
Solution:
A + B +C = 180
B + C = 180 - A
= 180 – 80
= 100
In triangle BDC,DBC+DCB +BDC= 180°
1
DBC = B (angular bisector)
2
1
DCB = C
2
1 1
In triangle BDC, B + C + BDC = 180°
2 2
1
= (100) + 𝐵𝐷𝐶 = 180°
2
BDC = 180 – 50
BDC = 130°

212. Simplify the equation Sin²x (1 + cot²x).


Solution:
sin²x + sin²x * cot²x
𝑐𝑜𝑠²𝑥
sin²x + sin²x *
𝑠𝑖𝑛²𝑥
sin²x + cos²x = 1

213. Assuming the earth to be a sphere of


radius 3960 mi, find the distance of point 36°N latitude from the equator.
Solution:
C = 2𝜋r
C =2 𝜋(3960)
C = 7920 𝜋
There are 360° in a circle so a degree of latitude is
1 = 7920 𝜋/ 360 = 22 𝜋, therefore the distance (d) in miles from the equator is
d = (22 𝜋)(36)
d = 2488 mi.
214. If sinxcosx + sin2x = 1, what are the
values of x in degrees?
Solution:

JAY T. OLIVEROS, REE, RME


Engineering Mathematics

Sinx cosx + sin2x = 1 (1)


Sinx cosx = sin2x
Sinx cosx = 0.5 sin2x (2)
Substitute (2) in (1)
0.5 sin 2x + sin 2x = 1
1.5 sin 2x = 1
Sin 2x = 0.6667
2x = 41.8
X = 20.9°
90° - 20.9° = 69.1°

215. If sin3x = cos6y then:

216. Evaluate 𝑐𝑜𝑡 −1 [2cos (𝑠𝑖𝑛−1 0.5)].


Solution:
𝑐𝑜𝑡 −1 [2 cos (𝑠𝑖𝑛−1 0.5)]
𝑐𝑜𝑡 −1 [2𝑐𝑜𝑠(30°)]
Cotx = 1.732
1
= 1.732
𝑡𝑎𝑛𝑥
1
tanx = = 0.5
1.732
−1
x = 𝑡𝑎𝑛 (0.5)
x = 30°

217. An airplane can fly at airspeed of 300


mph. if there is a wind blowing towards the east at 50 mph, what should be the planes
compass heading in order for its course to be 30 degrees. What will be the planes ground
speed if it flies at this course?
Solution:
By sine law
50 300
=
sin 𝛽 sin 60
𝛽= 8.3
α = 30° - Ɵ
α = 30° - 8.3 = 21.7°
ᴕ + 60° + 𝛃 = 180°
ᴕ + 60° + 8.3 = 180°
ᴕ = 111.7°

By sine law
sin 111.7° sin 8.3°
=
𝑉 50
V = 321.8 mph

218. From the given parts of spherical


triangle ABC, compute for angle A. (a = 120°, b = 73°15’, c = 62°45’)
Solution:
Using cosine law

JAY T. OLIVEROS, REE, RME


Engineering Mathematics

Cosb = cosa cosc + sina sinc cosB


Cos(73.25) = cos(120)cos(62.75) + sin(120) sin(62.75) cosB
B = 47.80
sin 𝐴 sin 47.80
=
sin(120) sin(73.25)
A = 42.06
A = 137.94 (its supplemented)
Or 137°56’

219. The diagonals of a parallelogram are 18


cm and 30 cm respectively. One side of a parallelogram is 12 cm. Find the area of the
parallelogram.
Solution:
1
ΔBEC = (15 + 9 + 12) = 18
2
A = 4√18(18 − 15)(18 − 9)(18 − 12)
A = 4√18(3)(9)(6)
A = 216 cm²

220. A quadrilateral has sides equal to 12 cm,


20 cm, 8 cm, and 17 cm respectively. If the sum of the two opposite angles is 225, find
the area of the parallelogram.
Solution:

𝑎+𝑏+𝑐+𝑑
S=
2
12+20+8+17
S= = 28.5
2
𝐴+𝐶 225
Ɵ= = = 112.5°
2 2
A = √(𝑠 − 𝑎)(𝑠 − 𝑏)(𝑠 − 𝑐 )(𝑠 − 𝑑 ) − 𝑎𝑏𝑐𝑑𝑐𝑜𝑠²Ɵ
(28.5 − 12)(28.5 − 20)(28.5 − 8)(28.5 − 17) − (12)
A=√
(20)(8)(17)𝑐𝑜𝑠²Ɵ
A = √(16.5)(8.5)(20.5)(11.5) − (4780.017)
A = 168.18m²

221. The sides of the cyclic quadrilateral are


a = 3 cm, b = 3 cm, c = 4 cm and d = 4
cm. Find the radius of a circle that can be inscribed in it.
Solution:
𝑎+𝑏+𝑐+𝑑
S=
2
3+3+4+4
S= =7
2

A = √(𝑠 − 𝑎)(𝑠 − 𝑏)(𝑠 − 𝑐)(𝑠 − 𝑑)


A = √(7 − 3)(7 − 3)(7 − 4)(7 − 4)
A = 12

JAY T. OLIVEROS, REE, RME


Engineering Mathematics

r=radius of the circle


Area of quadrilateral inscribed in a circle = r*s
12 = r (7)
r = 1.71 cm

222. How many diagonals can be drawn from


a 12 sided polygon?
Solution:
𝑛(𝑛−3)
d=
2
12 (12−3)
d=
2
d = 54

223. Find the area of


the regular polygon whose sides is 25m
and apothem is 17.2 m.
Solution:
1
A= 𝑏ℎ
2
1
A= (17.2)(25) =215
2
Area of pentagon = (215)(5)
= 1075
224. Find the area of a pentagon which is
circumscribing a circle having an area of 420.60 sq. cm.
Solution:
𝐴 = 𝜋𝑟 2
420.60 = 𝜋𝑎2
a = 11.57 cm

𝑠
a= 180
2tan ( )
𝑛
𝑠
11.57 = 180
2 tan( )
5
S = 16.81 cm
𝑝𝑒𝑟𝑖𝑚𝑒𝑡𝑒𝑟 (𝑎)
A of pentagon =
2
5 (16.81)(11.57)
=
2
= 486.23

𝜋 3𝜋
225. As x increases from to , |sin(2x)|
4 4
Solution:
As x changes g(x) = |sin(2x)| takes either positive values or zero
𝜋 𝜋 2𝜋
For x = ,g( ) = |sin( )| = 1
4 4 4
𝜋 𝜋 2𝜋
For x = , g( ) = |sin ( )|= 0
2 2 2
3𝜋 3𝜋 3𝜋
For x = , g( ) = |sin( )| = 1
4 4 4

JAY T. OLIVEROS, REE, RME


Engineering Mathematics

𝝅 𝟑𝝅
As x increases from to , |sin(2x)| decreases then increases.
𝟒 𝟒

226. If ax³ + bx² + cx + d is divided by x – 2


then the remainder is equal to
Solution:
Let P(x) = ax³ + bx² + cx + d. According
to remainder theorem, the remainder(r)
when dividing P(x) by x – 2 is equal to
P(2)
r = P(2) = a(2)³ + b(2)² + 2c + d
= 8a + 4b + 2c + d

227. A committee of 6 teachers to be formed


from 5 male teachers and 8 female teachers. If the committee is selected at random, what
is the probability that it has an equal number of male and female teachers?
Solution:
𝑛!
C(n,r) = (𝑛 − 𝑟)!
𝑟!
5! 8!
𝑐(5,3)𝑥 𝐶(8,3) ( )( ) 𝟏𝟒𝟎
3!2! 3!5!
P= = 13! =
𝑐(13,6) 𝟒𝟐𝟗
6!7!

228. The range of the function f(x) = -|x - 3|-


3 is
Solution:
f(x) = -|x - 3|- 3
|x - 2|> 0
-|x - 2| < 0
Subtract 3 from both sides
-x|-2|- 3 < -3
f(x) < -3
y < -3
229. What is the period of the function f(x) =
𝑝𝑖
3sin² (2x + )?
4
Solution:
𝑝𝑖
f(x) = 3sin² (2x + )
4
A sin (Bx + C) + D or A cos (Bx + C) + D
2𝑃𝑖
P=
|𝐵|
𝑝𝑖 1 1 𝑝𝑖
f(x) = 3sin² (2x + ) = 3[ − cos 2(2𝑥 + )]
4 2 2 4
Hence the period P of f(x) is given by
2𝑝𝑖 𝒑𝒊
P= =
|4| 𝟐

230. It is known that 3 out of 10 television


sets are defective. If 2 television sets are selected at random from the 10, what is the
probability that 1 of them is defective?

JAY T. OLIVEROS, REE, RME


Engineering Mathematics

Solution:
Probability that both sets picked are defective
3 2 6
𝑥 =
10 9 90
Probability that neither sets picked is defective
7 6 42
𝑥 =
10 9 90
Probability that exactly one set picked is defective
6 42 𝟕
1- - =
90 90 𝟏𝟓

231. In a triangle ABC, angle B has a size of


50°, angle A has a size of 32° and the length of side BC is 150 units. The length of side
AB is?
Solution:
B

b=150

A C

sin(32) sin (98)


=
150 𝑎
a = 280

232. For the remainder of the division of x³ -


2x² + 3kx + 18 by x – 6 to be equal to zero, k must be equal to?
Solution:
Using remainder theorem, the remainder (r) of division of P(x) by x – 6 is equal to P(6)
R = P(6) = (6)³ - 2(6)² - 3(6)k + 18
= 162 + 18k
162 + 18k = 0
18k = -162
k = -9
233. It takes pump (A) 4 hours to empty a
swimming pool. It takes pump (B) 6 hours to empty the same swimming pool. If the two
pumps are started together, at what time will the two pumps have emptied 50% of the
water in the swimming pool?
Solution:
RA = rate of A
RB = rate of B
t = time the two pumps will take to empty 50% of the water in the pool

1 1
RA = and RB =
4 6
1 1
t( + ) = 0.5
4 6
multiply all terms by LCM (12)
t(3 + 2) = 6

JAY T. OLIVEROS, REE, RME


Engineering Mathematics

5t = 6
6
t = = 1.2 hrs or 1hr and 12 minutes
5

234. The graph of r = 10 cos(Ɵ), where r and


Ɵ are the polar coordinates, is
Solution:
x = rcosƟ
x² + y² = r²cos²Ɵ
Multiply both sides by r
r² = 10rcosƟ
Substitute r² by x² + y² and rcos by x
x² + y² = 10x
x² - 10x + y² = o
(x – 5)² + y² = 25
The equation is a circle which gives a center of (5,0) and radius 5.

235. If (2 – i) x (a – bi) = 2 + 9i, where I is the


imaginary unit and a and b are real numbers, then a equals
Solution:
(2 – i) x (a – bi) = 2 + 9i
Expand the left side
2a – 2bi – ia – b = 2 + 9i
2a – b + i(-2b – a) = 2 + 9i
2a – b = 2 and -2b – a = 9
Subtract
[2a – b = 2] 2
4a – 2b = 4
-a – 2b = 9
5a = -5
a = -1

236. Lines L1 and L2 are perpendiculars that


intersect at the point (2, 3). If L1 passes through the point (0, 2) then L2 must pass through
the point
Solution:
m₁ of L₁
2−3 1
m₁ = =
0−2 2
m₁ x m₂ = -1
1
x m₂ = -1
2
m₂ = -2
Equation of L₂ is given by
y – 3 = -2(x – 2)
Line L₂ pass through the point (3,1)

237. In a plane there are 6 points such that


no three points are collinear. How many triangles do these points determine?

JAY T. OLIVEROS, REE, RME


Engineering Mathematics

Solution:
6!
₆C₃ = = 𝟐𝟎 𝒘𝒂𝒚𝒔
(3!)(3!)

238. In a circle with a diameter of 10 meters,


a regular five pointed star touching its circumference is inscribed. What is the area of the
part not covered by the star?
Solution:
A = area not covered by the star
As = area of the star
Ac = area of the circle
360
2Ɵ =
5
Ɵ = 36°
Ɵ
= 18°
2
Ɵ
Ɵ+ + α = 180°
2
𝛼 = 126°
By sine Law
𝑠𝑖𝑛18 𝑠𝑖𝑛126
=
𝑥 5
X = 1.91 m

A = Ac – As
1
𝐴 = 𝜋𝑟 2 – 10( )(5 x sinƟ)
2
1
A = 𝜋(5)2 - 10( )(5)(1.91)sin 36
2
A = 50.47 m²

239. Find the area of the hexagon with a


square having an area of 72 sq. cm inscribed in a circle which is inscribed in a hexagon.
Solution:
A of square = 72
Sides of square = √72
Diagonal of square = diameter of the circle
(6√2)(√2) = 12
12
Radius of the circle = =6
2
Side length of hexagon = side length of one equilateral triangles
2 12
= 6*( ) = = 4√3
√3 √3
𝑠²∗√3 (4√3)²√3
Area of hexagon = 6( )= 6( )
4 4
= 72√𝟑 or 124.71 cm²
240. The tangent and a secant are to a circle
from the same external point. If the tangent is 6 inches and the external segment of the
secant is 3 inches, compute the length of the secant.
Solution:

JAY T. OLIVEROS, REE, RME


Engineering Mathematics

UV² = UX*UY
(6)² = (3)(UY)
UY = 12

241. Two circles with radii 8 and 3 m are


tangent to each other externally. What is the distance between the points of tangency of
one of their common external tangencies?
Solution:

ΔDOQ, OD = 5 and OQ = 11
(DQ)² = (OQ)² - (OD)²
(DQ)² = (11)² - (5)² = 96
DQ = 4√𝟔 or 9.8 = CB

242. The diameters of the two circles that


are tangent internally are 18 and 8 respectively. What is the length of the tangent segment
from the center of the larger circle to the smaller circle?

243. Three identical circles are tangent to


each other externally. If the area of the curvilinear triangle enclosed between the points of
tangency of the 3 circles is 16.13 cm², compute the radius of each circle.

Solution:
Each sector is 60°
The circle is 360°
So the area of each circular sector is 1/6 of the whole circle area
𝜋
Acs = * r²
6
Since there are 3 circular sector within the equilateral triangle
𝜋
Acst = ∗ 𝑟²
2
So the area of curvilinear Δ is
Act = Aet – Acst
𝜋
Act = √3*r² - *r² = 16.13
2
𝜋
(√3 - )*r² = 16.13
2
16.13
r² =
0.1613
r² = 100.03
r = 10cm

244. A semi-circle of radius 14 cm is bent to


form a rectangle whose length is 1 cm more than its width. Find the area of the rectangle.
Solution:
𝜋*14 + 2*14 = perimeter of semicircle and rectangle
28 + 14𝜋 = perimeter
𝑊 𝑎𝑛𝑑 𝐿 = dimension of rectangle
2𝑊 + 2𝐿 = 28 + 14𝜋

JAY T. OLIVEROS, REE, RME


Engineering Mathematics

W + L = 14 + 7 𝜋
W + (W + 1) = 14 + 7 𝜋
2W + 1 = 14 + 7 𝜋
2W = 13 + 7 𝜋
13+7𝜋
W= = 17.4956
2

13+7𝜋
L= +1
2
L = 18.4956

A of rectangle = L x W
A = (18.4956)(17.4956)
A = 323.5816 cm²

245. A swimming pool is constructed in the


shape of two partially overlapping circles, each of radius 9 m. If the center each circle lies
on the circumference of the other, find the perimeter of the swimming pool.
Solution:
𝐴 = 𝜋𝑟 2
= 𝜋(9)2 = 254.47
𝐶 = 18𝜋
𝜋
Ɵ = radians
3
Area of shaded segment = r²(ƟsinƟ)/2
= 7.34 m
2
Length of major arc = 𝑐𝑖𝑟𝑐𝑢𝑚𝑓𝑒𝑟𝑒𝑛𝑐𝑒
3
2
= (18𝜋) = 12𝜋
3
Common Area = 2 x 7.34 = 14.68 m²
Area of the pool = 2(254.47 – 7.34) = 494.26 m²
Perimeter = 2 x 12 𝜋 = 75.4 m

246. The length of a side of a rhombus is 5


cm. If the shorter diagonal is of length 6 cm. What is the area of the rhombus?
Solution:

5
6

(5)² = (3)² + x²
X=4

Multiply 4 by 2 to get the full length of the 2nd diagonal


4 x 2 =8
1
Ar = (d1 * d2)
2

JAY T. OLIVEROS, REE, RME


Engineering Mathematics

1
Ar = (6 * 8)
2
Ar = 24 cm²

247. Two squares each of 12 cm sides


overlap each other such that the overlapping region is a regular polygon. Determine the
area of the overlapping region thus formed.
Solution:
The overlapped area is the area of a regular octagon with sides x√2 and
2x + x√2 = 12
12
x=
(2+√2)
x = 3.514
One side of the octagon is
S = x√2
S = (3.514)(√2) = 4.97
Area of a regular octagon is
A = (2 +2√2) S²
A = (2 + 2√2)(4.97)²
A = 119.27 cm²

248. The side of the regular pentagon is 25


cm. If the radius of the inscribed circle is 15 cm, find the area of the pentagon.
Solution:
1
Area of pentagon = (bh)(5)
2
1
= (25)(15)(5)
2
= 937.5 cm²

249. The capacities of two hemispherical


tanks are in the ratio 64: 125. If 4.8 kg of paint is required to paint the outer surface of the
smaller tank, then how many kilograms of paint would be needed to paint the outer surface
of the larger tank?
Solution:
Let Vs = volume of smaller tank
VL = volume of larger tank
Vps = volume of paint in smaller tank
VpL = volume of paint in larger tank
MpL = mass of paint needed in
larger tank
Vs/(VL )= 64/125
2
V = 𝜋𝑟 3
3
2
𝜋𝑟 3 64
3 𝑠
2 =
𝜋𝑟 3 125
3 𝐿
𝑟𝑠 64
( )³ =
𝑟𝑙 125

JAY T. OLIVEROS, REE, RME


Engineering Mathematics

𝑟𝑠 3 64
=√
𝑟𝑙 125
𝑟𝑠
= 0.8
𝑟𝑙
Vps 𝐴𝑆 𝑟𝑠
= = ( )²
VpL 𝐴𝐿 𝑟𝑙
4.8
= (0.8)2
MpL
MpL(0.8)² = 4.8
4.8
MpL =
(0.8)²
MpL = 4.5 kg

250. A wooden cone of altitude 10 cm is to


be cut into two parts of equal weight. How far from the vertex should the cut parallel to the
base be made?
Solution:
d³ = 10³
d³ = 1000
𝑑³ 1
Assuming a uniform density this ratio is also 1:2 so =
1000 2
d³ = 500
d= 7.937

251. A sphere of radius 5 cm and a right


circular cone of base radius 5 cm and a height 10 cm stand on a plane. Find the position
of a plane that cuts the two solids in equal circular sections.
Solution:
Let h be the height above the plane
The radius of the conic section relative to h will be

r = 5 – 5( )
10
the radius of the spherical section will be relative to h will be figured using pythagoras. As
the radius is 5, that will be the hypothenuse, the side not the radius will be length (5-h) so
r = √(52 − (5 − ℎ)²
r = √(25 − (25 − 10ℎ + ℎ2 )
r = √25 − 25 + 10ℎ − ℎ2
r = √10ℎ − ℎ²
√10ℎ − ℎ² = 5-5( ℎ )
10

√10ℎ − ℎ2 = 5 −
2
ℎ²
10h - h² = 25 – 5h +
4
5ℎ²
25 – 15h + =0
4
100 – 60h + 5h² = 0
20 – 12h + h²
Quadratic formula

JAY T. OLIVEROS, REE, RME


Engineering Mathematics

12±√(−122 −4(1)(20)
h=
2(1)

h = 10 or
h=2
as h = 10 is twice as high as the cone and at the upper vertex of the sphere, ignore this
solution so
h=2

252. A regular triangular pyramid has an


altitude of 9 m and a volume of 187. 06 cu. m. What is the base edge in meters?

Solution:
1
V = (𝐵𝑎𝑠𝑒 𝐴𝑟𝑒𝑎)ℎ
3
11
V = [ 𝑥² 𝑠𝑖𝑛Ɵ]ℎ
32
Ɵ = 60°
1 1
187.06 = ( ) 𝑥²𝑠𝑖𝑛60°(9)
3 2
X = 11.999 or 12

253. Two cylinders of equal radius 3m have


their axes at right angles. Find the volume of the common part.
Solution:
3
V = 2∫0 4 (32 − 𝑧 3 )𝑑𝑧
1
V = 8[9z - 𝑧³]30
3
1
V = 8[9(3) - )(3)³ - (0-0)]
3
V = 8(27 – 9)
V = 144 cm³

254. A solid has a circular base of radius 20


cm. Find the volume of the solid if every plane section perpendicular to a certain diameter
is an equilateral triangle.
Solution:
3
Height is given as h = b√
2
Circle about the origin having radius = 20 can be expressed as:
Y = √(400 − 𝑥 2 )
The vertical section triangle therefore has base b = 2y and the Area of triangle is A =
1 1 3
𝑏ℎ = 𝑏 (√ )
2 2 2

3 3
= (√ b² = (√ )(2y)² = (√3)𝑦² = (√3)(400 - x²)
4 4
Add all “dx” slices:
𝑥3
V= ∫[𝑎, 𝑏]𝐴(𝑥 )𝑑𝑥 = ∫[−20,20]√3(400 − 𝑥 2 )𝑑𝑥 = √3(400𝑥 − )|[−20,20]
3

JAY T. OLIVEROS, REE, RME


Engineering Mathematics

Since this is “odd” in x, just double the result from 0 to 20;


8000
V = 2√3 (8000- ) = 𝟏𝟖𝟒𝟕𝟓. 𝟐𝟎𝟖 𝐜𝐦³
3

255. If the edge of a cube is increased by


30%, by how much is the surface increased?
Solution:
𝐴₂ 𝑋2
= ( 1 )² (1)
𝐴₁ 𝑋
2
𝑋 = 1.3𝑋₁ (2)
Substitute (2) in (1)
𝐴₂ 1.3𝑋1
= ( 1 )²
𝐴₁ 𝑋
A₂ = 1.69 A₁
A₂ is increased by 69%

256. If the edge of a cube is decreased by x%,


its volume decreased by 48.8%. Find the value of x.
Solution:
Let each edge of cube be 100 units
Then its volume is 100³
= 1000000 cubic units
If the edge of cube decreased by x%
Then (100-x)³
Its volume decreased by 48.8%
= 512000
Then;
(100-x)³ = 512000
X = 20%

257. Find the acute angles between the two


planes 2x – y + z = 8 and x + y + 2z – 11 = 0.
Solution:
2x – y + z = 8
Magnitude = √(2)2 + (1)2 + (1)² = √6
x + y + 2z – 11 = 0
Magnitude = √(1)2 + (1)2 + (2)² = √6

(2𝑖−𝑗−𝑘)(𝑖+𝑗+2𝑘)
cosƟ = | |
(√6)(√6)
(2𝑥)+(−1 𝑥 1)+(1 𝑥 2)
=| |
6
1
cosƟ =
2
1
Ɵ = 𝑐𝑜𝑠 −1 ( )
2
Ɵ = 60°

258. Find the volume of the solid bounded

JAY T. OLIVEROS, REE, RME


Engineering Mathematics

by the plane x + y + z = 1 and the coordinate planes.


Solution:
x+y+z=1
Let y = 0 and z = 0
x=1
Vertex a = <1, 0, 0>
Y=1
Vertex b = <0, 1, 0>
Let x = 0 and y = 0
Z=1
Vertex c = <0, 0, 1>
1
V = . |𝑎. (𝑏 𝑥 𝑐)|
6
1 𝟏
V= ∗1∗1∗1=
6 𝟔

259. Compute the volume of a regular


icosahedron with sides equal to 6 cm.
Solution:
5(3+ √5
V= 𝑆3
12
5 (3+ √5)
V= (6)3
12
V = 471.2461 cm³

260. Compute the volume (in cm³) of a


sphere inscribe in an octahedron having sides equal to 18 cm.
Solution:
Let Vs = Volume of the sphere
18
r= (√6 )
6
r = 3√6
4
Vs = 𝜋𝑟 ³
3
4
Vs = = 𝜋(3√6)³
3
Vs = 1663.22 cm³

261. Find the volume of a spherical cone in a


sphere of radius 17 cm if the radius of its zone is 8cm.
Solution:

By Pythagorean Theorem
r² = R² - (h-R) ²

JAY T. OLIVEROS, REE, RME


Engineering Mathematics

(8)² = (17)² - (h – 17)²


h=2
2
V = 𝜋𝑅²ℎ
3
2
V = 𝜋(17)²(2)
3
V = 1210.56

262. A spherical wooden ball 15 cm in


diameter sinks to depth 12 cm in a certain liquid. Calculate the area exposed above the
liquid in cm².
Solution:
The area of spherical zone of one base is given by the formula
Z = 2𝜋RH
15
Z = 2𝜋 ( ) (12)
2
Z = 180𝜋
Area of spherical wooden ball = 4𝜋𝑟 2 = 4𝜋(7.5)2 = 225𝜋
Area exposed above the liquid = Area of the spherical wooden ball – Area of the spherical
zone of one base
= 225𝜋 − 180𝜋 = 𝟒𝟓𝝅

263. Given a solid right circular cone having


a height of 8 cm has a volume equal to 4 times the volume of the smaller cone that could
be cut from the same cone having the same axis. Compute the height of the smaller cone.
Solution:
1
V = 𝜋𝑟²ℎ (volume of right circular cone)
3
1 1
4 ( 𝜋𝑟 2 ℎ) = 𝜋𝑟²(8)
3 3
4 8
𝜋𝑟²ℎ 𝜋𝑟²
3 = 3
4 4
𝜋𝑟² 𝜋𝑟²
3 3
h = 2cm

264. The diameter of a sphere and the base


of a cone are equal. What percentage of that diameter must the cones height be so that
both volumes are equal?
Solution:
4
Volume of a sphere = 𝜋𝑟 ³
3
1
Volume of a cone = 𝜋𝑟 2 ℎ
3
4 ³ 1 2
𝜋𝑟 = 𝜋𝑟 ℎ
3 3

4
𝜋𝑟 ³
h = 13
𝜋𝑟 2
3
h = 4r
h = 2diameter

JAY T. OLIVEROS, REE, RME


Engineering Mathematics

The cone height needs to be twice the diameter that is 200% of the diameter.

265. The volume of a regular pyramid whose


base is a regular hexagon is 156 m³. If the altitude of the pyramid is 5m., find the sides of
the base.
Solution:
1
V = 𝜷𝒉
3
𝛃 = surface area of the hexagon
1
156 = 𝜷 (𝟓)
3
𝛃 = 93.6

1
𝛽
S = 34 √2
9
1
(93.6)
S = 3 √2 4
9
S=6m

266. The base of a cylinder is a hexagon


inscribed in a circle. If the difference in the circumference of the circle and the perimeter
of the hexagon is 4 cm. find the volume of the prism if it has an altitude of 20 cm.

267. The volume of a truncated prism with


an equilateral triangle as its horizontal base is equal to 3600 cm³. The vertical edges at
each corner are 4, 6, and 8 cm, respectively. Find one side of the base.

268. Aluminum and lead have specific


gravities of 2.5 and 16.48 respectively. If a cubical aluminum has edge of 0.30 m, find the
edge of a cubical block of lead having the same weight as the aluminum.
Solution:
Since the weight of aluminum and lead is equal, we can assume any number
Let x = edge of cubical block of Lead
By ratio and proportion of its density
Density = m/V where m = w/g

100 100
2.5 = 16.48
(0.30)³ 𝑥³
X = 0.16 m = 16 cm

269. Find the area of the pentagonal


spherical pyramid the angles of whose base are 105°, 126°, 134°, 146°, and 158° on the
sphere of radius 12 m.
Solution:

JAY T. OLIVEROS, REE, RME


Engineering Mathematics

A = [Ɵ – (n-2)𝜋]r²
𝜋
Ɵ=[105° + 126° + 134° + 146° + 158°]
180
223𝜋
Ɵ=
60
223𝜋
A=[ − (5 − 2) 𝜋](12)²
60
516𝜋
A=
5
A = 324.21

270. If the surface areas of two spheres are


24cm² and 96 cm² respectively. Find the ratio of their volume.
Solution:
SA = 4𝜋𝑟 2 SA = 4𝜋𝑟 2
24 = 4𝜋𝑟 2 96 = 4𝜋𝑟 2
r = 1.382 r = 2.764

4 4
V = 𝜋𝑟 3 V = 𝜋𝑟 3
3 3
4 3 4
V = 𝜋(1.382) V = 𝜋(2.764)3
3 3
V = 11 V = 88

11 𝟏
Ratio of their volumes = 𝑜𝑟
88 𝟖

271. Considering the earth as a sphere of


radius 6400 km, find the radius of the 60th parallel of latitude.
Solution:
Let r = radius of the parallel latitude
re = 6400 km (radius of the earth)
𝑟
cos 60 =
𝑟𝑒
r = re cos 60
r = 6400 cos 60
r = 3,200 km

272. A conical vessel has a height of 24 cm.


and a base diameter of 12 cm. It holds water to a depth of 18 cm above its vertex. Find
the volume of its content.
Solution:
By ratio and proportion
6 𝑥
=
24 18
X = 4.5

4
V = 𝜋𝑥 2 ℎ
3
4
V = 𝜋(4.5)2 (18)
3
V = 381.70 cm³

JAY T. OLIVEROS, REE, RME


Engineering Mathematics

273. A sphere is dropped in a can partially


filled with water. What is the rise in height of the water if they have equal diameter?
Solution:
Vcan = Vsphere
4
𝜋𝑟 2 ℎ 𝜋𝑟 3
3
=
𝜋𝑟 2 𝜋𝑟 2
4
h= r
3
h = 1.33r
1.33𝑑
h=
2
h = 0.67d

274. A wooden cone is to be cut into two


parts of equal volume by a plane parallel to its base. Find the ratio of the heights of the
two parts.
Solution:
A

h V₁
D O₁ E H
V₂ r₂ H-h
B 0 C
r₁
Let ABC the right wooden cone and cut by the plane parallel to BC
BO = r₁, DO₁ = r₂ AO = H AO₁ = h
In Δ AOD and Δ AOB
1 = 1, 2 = 3
𝐴𝑂₁ 𝐷𝑂₁
=
𝐴𝑂 𝐵𝑂
ℎ 𝑟₂
=
𝐻 𝑟₁
Let V₁ = volume of the small cone
V₂ = volume of the frustum
V = Volume or larger cone
Given that V₁ = V₂
But V₁ + V₂ = V
= 2V₁ = V
1 1
2* 𝜋𝑟22 ℎ = 𝜋𝑟12 𝐻
3 3
2𝑟22 ℎ = 𝑟12 𝐻
𝑟12 𝐻
2= *
𝑟22 ℎ
𝑟₁ 𝐻
2 = ( )²( )
𝑟₂ ℎ
𝐻 𝐻
2 = ( )² ( )
ℎ ℎ
𝐻
2 = ( )³

JAY T. OLIVEROS, REE, RME


Engineering Mathematics

1
𝐻
23 =

1
𝐻
2 - 1=
3 −1

1
𝐻−ℎ
2 - 1=
3

1 𝐻−ℎ
1 =

23 − 1
= 3.847 (ratio of their heights)

275. The ratio of the area of regular polygon


circumscribed in a circle to the area of inscribed regular polygon of the same
numberofsidesis4:3. Find the number of sides.
Solution:
9 𝜋 2
𝜋𝑟 2 = 𝜋 ( 𝑐𝑜𝑠𝑒𝑐 )
2 𝑛
2 𝜋9² 𝜋
𝜋𝑟 = ( 𝑐𝑜𝑠𝑒𝑐² )
4 𝑛
2 9 𝜋 2
𝜋𝑟 = 𝜋 ( 𝑐𝑜𝑡 )
2 𝑛
2 𝜋9² 𝜋
𝜋𝑟 = ( 𝑐𝑜𝑡² )
4 𝑛
Common ratio:
𝜋9² 𝜋
𝑐𝑜𝑠𝑒𝑐² 4
4 𝑛
𝜋9² 𝜋 =
𝑐𝑜𝑡² 3
4 𝑛
1
𝜋
𝑠𝑖𝑛² ( )
𝑛 4
𝜋 =
cos ( )
𝑛 3
𝜋
𝑠𝑖𝑛² (𝑛)
𝜋
4 𝑐𝑜𝑠² ( ) 3
𝑛
=
4 4
𝜋 √3 𝜋
cos = =
𝑛 2 6
n=6

276. A rectangle ABCD which measures 18


by 24 cm is folded once, perpendicular to diagonal AC, so that the opposite vertices A and
C coincide. Find the length of the fold.
Solution:
1
A = 𝜋(15)2 − 180 = 173.43 𝑐𝑚²
2
d = √(18)2 + (24)² = 30 cm
x² = (18)² + (24 – x)²
x² = 324 + 576 – 48x + x²
48x = 325 + 576
X = 18.75 cm
𝑑
x² = ( )² + 𝑦²
2
(18.75)² = (15)² + y²
y = 11.25

JAY T. OLIVEROS, REE, RME


Engineering Mathematics

Length of fold = 2y = 2(11.25) = 22.5 cm

277. If the straight lines ax + by + c = 0 and


bx + cy + a = 0 are parallel, then which of the following is correct?
Solution:
Express the two lines in standard format that is y = mx + c
ax + by + c = 0
by = -ax – c
−𝑎𝑥 𝑐
y= −
𝑏 𝑏

bx + cy + a = 0
cy = -bx – a
−𝑏𝑥 𝑎
y= −
𝑐 𝑐
Given both lines are parallel implies their slopes are equal that is:
𝑎 𝑏
(-- ) = (− )
𝑏 𝑐
𝑎 𝑏
= = 𝒃² = 𝒂𝒄
𝑏 𝑐

278. Find the equation of the perpendicular


bisector of the segment joining the points (2, 6) and (-4, 3).
Solution:
𝑦₂−𝑦₁
m=
𝑥₂−𝑥₁

3−6 −3 1
m= = =
−4−2 −6 2

We need a line perpendicular to this line so,


1
𝑥 𝑚 = −1
2
m = -2
𝑥₁+𝑥₂ 𝑦₁+𝑦₂
( ), ( )
2 2
2−4 6+3
( ), ( )
2 2
9
(-1, ) = midpoint which the 2nd line passes
2
y - y₁ = m(x - x₁)
9
y – = −2(𝑥 + 1)
2
9
y - = −2𝑥 − 2
2
9
2x + y - + 2 = 0
2
5
2 [2x + y - = 0] 2
2
4x + 2y – 5 = 0

279. The vertices of the base of an isosceles

JAY T. OLIVEROS, REE, RME


Engineering Mathematics

triangle are (-1,-2) and (1, 4). If the third vertex lies on the line 4x + 3y = 12, find the area
of the triangle.
Solution:

d₂ d₃
h

B d₁ C
(-1,-2) (1,4)

4x + 3y = 12
4x + 3(0) = 12
X=3
4(3) + 3y = 12
3y = 12-12
Y=0
Point A = 3, 0
d₁ = √(1 + 1)2 + (4 + 2)² = 2√10
d₂ = √(−1 − 3)2 + (−2 − 0)² = 2√5
d₃= √(1 − 3)2 + (4 − 0)² = 2√5

(2√5)² = (√10)²+ h²
h = 3.162
1
A = 𝑏ℎ
2
1
A = (2√10)(3.162)
2
A = 10

280. The coordinates of the two vertices of a


triangle are (6,-1) and (-3,7). Find the coordinates of the third vertex so that the centroid
of the triangle will be at the origin.
Solution:
Let the coordinates of the third vertex be (x,y)
Then the equation for x and y are:
6 + (-3) + x = 0
X = -3
(-1) + 7 + y = 0
y = -7 + 1
y = -6
coordinates of 3rd vertex = (-3, -6)

281. Compute the angle between the line 2y

JAY T. OLIVEROS, REE, RME


Engineering Mathematics

– 9x – 18 = 0 and the x-axis.


Solution:
2y – 9x – 18 = 0
2y = 9x + 18
9
y= 𝑥+9
2
9
m=
2
9
−1 (
𝑡𝑎𝑛 ) = 𝟕𝟕. 𝟒𝟕
2
282. Find the value of k if the y-intercept of
the lines 3x – 4y – 8k = 0 is equal to 2.
Solution:
3x – 4y -8k = 0
3x = 8k + 4(2)
3x = 8k + 8
8𝑘+8
X=
3

-4y = -3x + 8k
3𝑥 8𝑘
y= −
4 4
3
y = 𝑥 − 2𝑘
4
Substitute the value of x and y
3 8𝑘+8
2= ( ) − 2𝑘
4 3
k = -1

283. Find the area of the polygon whose


vertices are (2,-6),(4,0),(2,4),(-3,2),(-3,3).

284. In the triangle ABC having vertices at


A(-2,5), B(6,1)and C(-2,-3) find the length of the median from the vertex B to side AC.
Solution:
A

B C

−2−2 −3+5
D=( ), ( )
2 2
D = -2, 1

d = √(6 + 2)2 − (1 − 1)²


d = √64 = 8

285. A line has an equation 3x – ky – 8 = 0.


Find the value of k if this line makes an angle of 45° with the line 2x + 5y – 17 = 0.
Solution:
JAY T. OLIVEROS, REE, RME
Engineering Mathematics

2x + 5y – 17 = 0
5y = -2x + 17
−2 17
Y= 𝑥+
5 5

−2
m₁=
5

3x – ky – 8 = 0
-ky = -3x + 8
3
Y= 𝑥−8
𝑘

3
m₂ =
𝑘
𝑚₁−𝑚₂
tanƟ = | |
1+𝑚₁𝑚₂

−2 3

5 𝑘
tan45° = | −2 3 |
1+( )( )
5 𝑘

2𝑘−15
1=| −6 |
1+
5𝑘
6
2k – 15 = 1-
5𝑘
k= 7.924

286. The points (1,3) and (5,5) are two


opposite vertices of a rectangle. The other two vertices lie on the line 2x – y + k = 0. Find
the value of k.
Solution:
Points (1,3) and (5,5) forms diagonal of the rectangle. The midpoint will lie on the other
diagonal
1+5 3+5
( , ) = 3,4
2 2

2x – y + k = 0
2(3) -4 + k = 0
6–4+k=0
k = -2

287. Let m1 and m3 be the respective slopes


of two perpendicular lines. Then,

For perpendicular lines;


−1
m₁ =
𝑚₂
m₁ x m₂ = -1

288. The abscissa of a point is 3. If its


distance from a point (8,7) is 13, find its ordinate.

JAY T. OLIVEROS, REE, RME


Engineering Mathematics

Solution:

13 = √(8 − 3)2 + (7 − 𝑦)2


y = -5

13 = √(3 − 8)2 + (𝑦 − 7)2


y = 19

289. If the points (-3, -5), (p,q) and (3,4) lie


on a straight line, then which of the following is correct?
Solution:
Point 1 (-3,-5) p₁ = −3 q₁ = -5
Point 2 (3, 4) p₂ = 3 q₂ = 4

Using 2 point form


𝑞₂−𝑞₁
q - q₁= (p - p₁)
𝑝₂−𝑝1

4+5
q+5= (p + 3)
3+3

9
q + 5 = (p + 3)
6
3
q+5= ( 𝑝 + 3)
2
3𝑝+6
q+5=
2
3p – 2q = 6 – 5
3p – 2q = 1

290. Find the equation of the line parallel to


7x + 2y – 4 = 0 and passing through (-3,-5).
Solution:
7x + 2y – 4 = 0
2y = -7x + 4
−7
y= 𝑥+2
2
−7
m=
2

(y - y₁) = m (x - x₁)
−7
y+5= (x + 3)
2
−7 21
y+5= 𝑥−
2 2
7 21
𝑥+𝑦+ +5=0
2 2
7 31
2[ 𝑥 + 𝑦 + = 0]2
2 2
7x + 2y + 31 = 0

291. Find the area of the triangle whose

JAY T. OLIVEROS, REE, RME


Engineering Mathematics

vertices are (1,1), (3,-3) and (5,-3).


Solution:
Plot the points
1
A = 𝑏ℎ
2
1
A= (2)(4)
2
8
A= =𝟒
2

292. Determine the x – intercept of the line


passing through (4,1) and (1,4).
Solution:
Using two point form
𝑦₂−𝑦₁
y - y₁ = (x - x₁)
𝑥₂−𝑥₁

1−4
y–4= (𝑥 − 1)
4−1
y – 4 = -x + 1
x+y=5
Substitute y = 0 to solve for x-intercept
x+y=5
x+0=5
x=5

293. Find the slope of the line having a


parametric equation of x = 2 + t and y = 5 – 3t.
Solution:
x=2+t
t=x–2 eq.1
y = 5 – 3t eq.2

Substitute eq. 1 in eq.2


y = 5 – 3t
y = 5 – 3(x – 2)
y = 5 – 3x + 6
y = -3x + 11
slope = -3

294. The midpoint of the line segment


joining a moving point to (6,0) is on the line y = x. Find the equation of its locus.
Solution:
Since M is the midpoint then,
𝑥+6 𝑦+0
( , )
2 2
𝑥+6 𝑦
= ,
2 2
Satisfy the equation y = x
𝑦 𝑥+6
2[ = ]2
2 2

JAY T. OLIVEROS, REE, RME


Engineering Mathematics

y= x + 6
x–y+6=0

295. The base of an isosceles triangle is the


line from (4,-3) to (-4,5). Find the locus of the third vertex.
Solution:
(x–4)²+(y+3)²-(x+4)²-(y-5)² = 0
-8x-8x+6y+10y+16+9-16-25 = 0
-16x + 16y – 16 = 0
x – y + 1 = 0 (locus of the 3rd vertex)
296. What is the new equation of the line
5x + 4y + 3 = 0 if the origin is translated to the point (1,2)?
Solution:
5x + 4y + 3 = 0
5(x + 1) + 4(y + 2) + 3 = 0
5x + 5 + 4y + 8 + 3 = 0
5x + 4y + 16 = 0

297. One end of the diameter of the circle


(x – 4)² + y² = 25 is the point (1,4). Find the coordinates of the other end of this diameter.
Solution:
(x – 4)² + y² = 25
h=4 k=0 r=5
Center = 4, 0
One end of the diameter = (1, 4)
Plot the points
Draw a horizontal line from (1, 4) to a point directly above the center, then vertical line
from there to the center (4, 0). The horizontal line is 3 units long and the vertical line is 4
units long. Then, do the same from the center of the circle 3 units right from (4, 0) then 4
units downward and you end up at the point (7, -4)

298. Determine the area bounded by the


curve x² + y² - 6y = 0.

Solution:
x² + y² - 6y = 0
By completing the square:
x² + y² - 6y + (3)² = 3²
x² + (y – 3)² = 3²

Standard equation of a circle with


center at (h,k)
(x – h)² + (y – k)² = r²
Radius = 3

𝐴 = 𝜋𝑟 2
A = 𝜋(3)2

JAY T. OLIVEROS, REE, RME


Engineering Mathematics

A = 28.27 square units

299. How far is the center of the circle x² +



- 10x – 24y + 25 = 0 from the line y = 2?
Solution:
x² + y² - 10x + 25 = 0
By completing the square to find the center and the radius
(x – 5)²+25+(y – 12)²+144=-25+25+144
(x – 5)²+25 + (y – 12)²+144 = 144
h, k = (5,12)
radius = 12
Distance from Line y = 2
To find the distance between the center of the circle and the line, just add the y-coordinates
to the radius of the circle, therefore,
d=r+2
d = 12 + 2
d = 14

300. Find the equation of the circle tangent


to the y-axis and the center is at (5,3).
Solution:
General formula
(x – h)² + (y – k)² = r²
Since the circle touches y – axis at only one point and from the center to the y axis distance
on the x-axis is 5 units, therefor r = 5
(x – 5)² + (y – 3)² = 5²
(x – 5)² + (y – 3)² = 25

301. Find the equation of the circle


circumscribing a triangle whose vertices are (0,0), (0,5) and (3,3).
Solution:
Gen Eq. = x² + y² + Dx + Ey + F = 0

(0,0)=0²+0²+D(0)+E(0)+F = o
F = 0 Eq.1

(0,5)=0²+5²+D(0)+E(5)+F = 0
25 + 5E + F = 0 Eq. 2

(3,3)=3²+3²+D(3)+E(3)+F = 0
9+9+3D+3E+F = 0
18 + 3D + 3E + F = 0 Eq. 3

Subtract Eq.2 from Eq.1


F – (25 + 5E + F) = 0
−5𝐸 25
=
5 −5

JAY T. OLIVEROS, REE, RME


Engineering Mathematics

E = -5 Eq.4

Subtract Eq. 3 From Eq. 2


25 + 5E + F – (18 + 3D + 3E + F) = 0
7 + 2E – 3D = 0
7 + 2E = 3D Eq. 5

Substitute the value of E in Eq. 2


25 + 5E + F = 0
25 + 5(-5) + F = 0
25 – 25 + F = 0
F=0

Substitute the value of E in Eq. 5


7 + 2E = 3D
7 + 2(-5) = 3D
-3 = 3D
D = -1

x² + y² + Dx + Ey + F = 0
x² + y² - x – 5y = 0

302. A parabola having its axis along the x-


axis passes through (-3,6). Compute the length of the latus rectum if the vertex is at the
origin.
Solution:
a = -3-0
a = -3
latus rectum = 4a
= 4(3)
= 12
,
303. A hut has a parabolic cross-section
whose height is 30 m and whose base is 60 m wide. If the ceiling 40 m is to be placed
inside the hut, how high will it be above the base?
Solution:
Representing a hut, draw a parabola that opens downward with the vertex at (0,20)
Standard Form of Equation
y = A(x – h)² + k
(h , k) = (x,y) coordinates of the vertex
y = A(x – 0)² + 30
y = Ax² + 30
Using coordinates of right side of the base (30, 0 ) find A
0 = A(30)² + 30
0 = 900A + 30
-30 = 900A
−1
A=
30

JAY T. OLIVEROS, REE, RME


Engineering Mathematics

If the ceiling 40 m is to placed inside the hut, x = 20


−1
Y = ( ) (20)2 + 30
3
Y = 16.67 m

304. Find the coordinates of the focus of the


parabola x² = 4y – 8.
Solution:
x² = 4y – 8
Write the equation in standard form
4p(y – k) = (x – h)²
4.1. (y – 2) = (x – 0)²
(h, k) = (0, 2)
P=1
Coordinates of the focus
= (0, 2 + p)
= (0, 2+1)
= (0, 3)

305. An ellipse has an eccentricity of 1/3.


Compute the distance between directrices if the distance between foci is 4.
Solution:
2ae = 4
1
2a( ) = 4
3
2a = 12
a=6
distance between directrices
2𝑎 2(6)
=
𝑒 1
3
d = 12(3) = 36

306. An ellipse has a length of semi-major


axis of 300 m. Compute the second eccentricity of the ellipse.

307. Compute the circumference of an


ellipse whose diameters are 14 and 10 meters.

JAY T. OLIVEROS, REE, RME


Engineering Mathematics

Solution:
𝑎²+𝑏²
2𝜋√ +𝜋 (𝑎+𝑏)
𝑒
C=
2
7²+5²
2𝜋√ +𝜋 (7+5)
𝑒
C=
2
C = 37.95
= 38

308. Find the eccentricity of a hyperbola


having distance between foci equal to 18 and the distance between directrices equal to 2.
Solution:
Distance between directrices: 2a/e
Distance between focci: 2ae

2𝑎𝑒 18
=
2𝑎 2
( )
𝑒
𝑒2 = 9
e = √9
e=3

309. Find the length of the tangent from


point (7,8) to the circle x² + y² - 9 = 0.
Solution:

OP = √(7)2 + (8)2
OP = √113
OP = 10.63
QP = √10.632 − 32
QP = 10.2

310. What is the equation of the equation of


the directrix of the parabola y² = 16x?
Solution:
𝑦 2 = 16𝑥
𝑦 2 = 4𝑎𝑥 → 𝑠𝑡𝑎𝑛𝑑𝑎𝑟𝑑 𝑒𝑞𝑢𝑎𝑡𝑖𝑜𝑛
16 = 4a
a=4

Equation of directrix
X = -a
X = -4

311. Find the radius of the circle 2x² + 2y² -


3x + 4y – 1 = 0.

JAY T. OLIVEROS, REE, RME


Engineering Mathematics

Solution:
2x² + 2y² - 3x + 4y – 1 = 0
Simplify
3 1
x² +y² - 𝑥 + 2𝑦 − =0
2 2
3 1
x²- 𝑥 + 𝑦² + 2𝑦 =
2 2
3 9 1 9
x²- 𝑥 + + 𝑦² + 2𝑦 + 1 = + +1
2 16 2 16
3 33
(x - )² + (𝑦 + 1)2 =
4 16
33
r=√
16
√𝟑𝟑
r=
𝟒

312. Find an equation for the hyperbola with


foci at (1,3) and (9,3), and eccentricity 2.
Solution:
1+9 3+3
h,k = ( , )
2 2
h,k = (5, 3)
(𝑥 − ℎ)² (𝑦 − 𝑘)2
− =1
𝑎² 𝑏2
(𝑥 − 5)² (𝑦 − 3)2
− =1
4 12
2 2
𝑥 − 10𝑥 + 25 𝑦 − 6𝑦 + 9
[ − = 1] 12
4 12
3x² - 30x + 75 - y² + 6y – 9 = 12
3x² - y² - 30x + 6y + 54 = 0

313. Find the equation of the locus of a


point which moves so that its distance from (1,-7) is always 5.
Solution:
(x – 1)² + (y + 7)² = 5²
x² - 2x + 1 + y² + 14y + 49 = 25
x² + y² - 2x + 14y + 50 – 25 = 0
x² + y² - 2x + 14y + 25 = 0

314. The difference of the distance of a


moving point from (1,0) and (-1,0) is 1. Find the equation of its locus.

Solution:
√(𝑥 − 1)2 + (𝑦 − 0)² − √(𝑥 + 1)2 + (𝑦 − 0)2 = 1

√𝑥² − 2𝑥 + 1 + 𝑦²= √𝑥² + 2𝑥 + 1 + 𝑦² + 1


(√𝑥² − 2𝑥 + 1 + 𝑦²)² = (√𝑥² + 2𝑥 + 1 + 𝑦² + 1)²
𝑥² − 2𝑥 + 1 + 𝑦²= 𝑥² + 2𝑥 + 1 + 𝑦² + 2√𝑥² + 2𝑥 + 1 + 𝑦² + 1

JAY T. OLIVEROS, REE, RME


Engineering Mathematics

2
(−4𝑥 − 1)2 = (2√𝑥 2 + 2𝑥 + 1 + 𝑦 2 )
16𝑥² + 8𝑥 + 1 = 4(𝑥 2 + 2𝑥 + 1 + 𝑦 2 )
16𝑥² + 8𝑥 + 1 = 4𝑥² + 8𝑥 + 4 + 4𝑦²
12x² - 4y² - 3 = 0
12x² - 4y² = 3

315. A circle has its center on the line 2y = 3x


and tangent to the x-axis at (4,0). Find the radius.
Solution:
2y = 3x
The line tangent to it which could identified as x = 4
Coordinates = (4, 0)
2y = 3(4)
2y = 12
Y=6

Circle Equation
(x – h)² + (y – k)² = r²
(x – 4)² + y² = 36
r = √36
r=6

316. Find the shortest distance from (3,8) to


the curve x² + y² + 4x – 6y = 12.
Solution:
x² + y² + 4x – 6y = 12
(x + 2)² + (y – 3)² = 12 + (2)² + (3)²
(x + 2)² + (y – 3)² = 25
C = (-2,3) r=5

Solving the distance from (3,8)to (-2,3)


d = √(𝑥₂ − 𝑥₁)² + (𝑦₂ − 𝑦₁)²
d = √(3 + 2)2 + (8 − 3)2
d = 7.07
shortest distance = d – r
= 7.07 – 5
= 2.07
317. The focus of the hyperbola y² = 4x is at:
Solution:
The equation of parabola with vertex at origin is given by
y² = 4ax
its vertex is (0,0)
its focus is (a,0)
y² = 4x
4𝑎𝑥 4𝑥
=
4 4𝑥

JAY T. OLIVEROS, REE, RME


Engineering Mathematics

a=1
Therefore the focus of hyperbola = (1,0)

318. An arc in the form of a parabola is 60 m


across the bottom. The highest point is 16 m above the horizontal base. What is the length
of the beam placed horizontally across the arc 3 m below the top?
Solution:
Three points (-30, 0), (0, 16) @ (30, 0) @ (0, 16) is the vertex
Y = a(x – 0)² + 16
Y = ax² + 16
0 = a(30)² + 16
900𝑎+16
0=
4
0 = 225a + 4
−4
=𝑎
225
Y = 16 – 3 = 13
−4
Y= 𝑥² + 16
225
−4
13 = 𝑥² + 16
225

225
√3 ( )=𝑥
4
X = 12.99 x 2
X = 25.98

319. A curve has an equation of x² = cy + d.


the length of latus rectum is 4 and the vertex is at (0,2). Compute the value of C and d.
Solution:
Vertex = (0,2)
Latus rectum = 4
(y – k)² = 4a (x – h)
Substitute the vertex (h,k)
(y – 2)² = 4a (x – 0)
Distance between the vertex and latus rectum = 2
(y – 2)² = 4(2)(x – 0)
(y – 2)² = 8(x – 0)
C = 4 d = -8

320. What conic section is 2x² - 8xy + 4x =


12?
Ans: Hyperbola

321. What conic section is described by the


equation r = 6/(4 –3 cosƟ)?
Ans: Ellipse

322. An ellipse has its center at (0,0) with its

JAY T. OLIVEROS, REE, RME


Engineering Mathematics

axis horizontal. The distance between the vertices is 8 and its eccentricity is 0.5. Compute
the length of the longest focal radius from point (2,3) on the curve.
Solution:
Vertex = (0,4)
Point (2,3)
L = (0-2)² + (4 – 3)²
L=5

323. Determine the equation of the common


tangents to the circles x² + y² + 2x + 4y – 3 = 0 and x² + y² - 8x – 6y + 7 = 0.

324. An arc in the form of a parabolic curve


is 40 m across the bottom. A flat horizontal beam 26 m long is placed 12 m above the
base. Find the height of the arc.
Solution:
Plot the points (-20,0) and (20,0) for the bottom
2 other points are (-13, 12) and (13,12)
The form is y = -ax² + c
0 = -a(20)² + c
0 = -400a + c
12 = -a(13)² + c
12 = -169a + c
Subtract
0 = -400a + c
12 = -169a + c
-12 = -231a

44
a=
7
44
0 =-( )(400) + 𝑐
7
−1600
0= +𝑐
77
C = 20.78 m → height of the arc

325. Evaluate: Lim (tan³3x)/x³ as x


approaches 0.
Solution:
𝑡𝑎𝑛3 3𝑥
lim x→₀ ( ) = 𝑑𝑖𝑣𝑒𝑟𝑔𝑒𝑠
𝑥³

326. Evaluate the integral xcosxdx.


Solution:
∫ 𝑢𝑣 ′ 𝑑𝑥 = 𝑢𝑣 − ∫ 𝑢𝑣 ′ 𝑑𝑥
∫ 𝑥 cos 𝑥 𝑑𝑥
Let u = x

JAY T. OLIVEROS, REE, RME


Engineering Mathematics

u’ = 1
v’ = cos x
v = sinx
then; ∫ 𝑥 cos 𝑥 𝑑𝑥 = 𝑥 sin 𝑥 − ∫ 1. 𝑠𝑖𝑛𝑥𝑑𝑥
x sin x – (-cos x)
= x sin x + cos x
= x sin x + cos x + c

327. Find the limit: sin2x/sind3x as x


approaches to 0.
Solution:
Lim
𝑠𝑖𝑛2𝑥
𝑠𝑖𝑛3𝑥
x→₀

Lim
𝑠𝑖𝑛2𝑥
2𝑥 2𝑥
𝑠𝑖𝑛3𝑥 𝑥
3𝑥
3𝑥
x→₀
𝑠𝑖𝑛2𝑥
lim 2x→₀ 2
2𝑥
𝑠𝑖𝑛3𝑥 x
lim 3𝑥→₀ 3
3𝑥
𝟐
=
𝟑

328. A snowball is being made so that its


volume is increasing at the rate of 8 ft³/min. Find the rate at which the radius is increasing
when the snowball is 4 ft in diameter.
Solution:
𝑑𝑣
=8
𝑑𝑡
𝑑𝑟
Find 𝑤ℎ𝑒𝑛 𝑟 = 2
𝑑𝑡
4
Equation V = 𝜋𝑟 3
3
𝑑𝑣 𝑑𝑟
= 4𝜋𝑟 2
𝑑𝑡 𝑑𝑡
𝑑𝑟
8 = 4 𝜋(2)2
𝑑𝑡
𝑑𝑟 8 1
= =
𝑑𝑡 4𝜋 2𝜋
= 0.159 ft/min

329. A stone is dropped into a still pond.

JAY T. OLIVEROS, REE, RME


Engineering Mathematics

Concentric circular ripples spread out, and the radius of the disturbed region increases at
the rate of 16 cm/s. At what rate does the area of the disturbed increase when its radius
is 4 cm?
Solution:
Rate of change in radius = 10 cm/s
𝑑𝐴
Rate of change of Area =
𝑑𝑡
𝐴 = 𝜋𝑟 2
𝑑𝐴 𝑑𝑟
= 2 𝜋r( )
𝑑𝑡 𝑑𝑡
𝑑𝐴
= 2𝜋(4)(16)
𝑑𝑡
= 128𝜋
= 𝟒𝟎𝟐. 𝟏𝟐 𝐜𝐦²/𝐬

330. Find the limit (x + 2)/(x – 3) as x


approaches 3.
Since the function approaches – infinity from left to right, the limit does not exist.
Ans.: Infinity

331. A man 1.8 m tall is walking at the rate


of 1.2 m/s away from a lamp post 6.7 m high. At what rate is the tip of his shadow receding
from the lamp post?
Solution:
6.7 1.8
=
𝑦 𝑦−𝑥
6.7y – 6.7x = 1.8y
4.9y = 6.7x
y = 1.367x
y = 1.367 (1.2)
y = 1.64 m/s

332. A man on a wharf is pulling a rope tied


to a raft at a rate of 0.6 m/s. If the hands of the man pulling the rope are 3.66 m above the
water, how fast is the raft approaching the wharf when there are6.1 m of the rope out?
Solution:
3.66² + x² = y²
𝑑𝑥 𝑑𝑦
0 + 2x = 2𝑦 𝑤ℎ𝑒𝑟𝑒
𝑑𝑡 𝑑𝑡
𝑥 = √6.1² − 3.66² = 4.88
𝑑𝑦
= −0.6
𝑑𝑡
𝑑𝑥 𝑦𝑑𝑦
=
𝑑𝑡 𝑥𝑑𝑡
𝑑𝑥 6.1
= (−0.6)
𝑑𝑡 4.88
= −𝟎. 𝟕𝟓 𝐦/𝐬

333. Evaluate the limit : tanx/x as x

JAY T. OLIVEROS, REE, RME


Engineering Mathematics

approaches 0.
Solution:
𝑡𝑎𝑛𝑥
Lim
𝑥
x→₀
𝑠𝑖𝑛𝑥/𝑐𝑜𝑠𝑥
lim
𝑥
x→₀
𝑠𝑖𝑛𝑥
lim ( )/(𝑐𝑜𝑠𝑥)
𝑥
x→₀
𝑠𝑖𝑛𝑥 1
lim ( ) .( )
𝑥 𝑐𝑜𝑠𝑥
x→₀
𝑠𝑖𝑛𝑥 1
(lim x→₀ ) . (lim 𝑥 →0 )
𝑥 𝑐𝑜𝑠𝑥
1
=1x
𝑐𝑜𝑠Ɵ
=1

334. A man is riding his car at the rate of 30


km/hr toward the foot of a pole 10 m high. At what rate is he approaching the top when he
is 40 m from the foot of the pole?
Solution:
𝑑𝑥
= 30 𝑘𝑚/ℎ𝑟
𝑑𝑡
Where 30 km/hr = 8.33 m/s
𝑑𝑥
= −8.33 𝑚/𝑠
𝑑𝐿
x² + 10² = y²
y² = 40² + 10²
y = 41.23

2𝑥𝑑𝑥 2𝑦𝑑𝑦
=
𝑑𝑡 𝑑𝑡
𝑥𝑑𝑥 𝑦𝑑𝑦
=
𝑑𝑡 𝑑𝑡
𝑑𝑦 𝑑𝑥 𝑥
= ( )
𝑑𝑡 𝑑𝑡 𝑦
𝑑𝑦 −8.33(40)
=
𝑑𝑡 41.23
𝒅𝒚
= −𝟖. 𝟎𝟖 𝒎/𝒔
𝒅𝒕

335. Find the point on the curve y = x³ at


which the tangent line is perpendicular to the line 3x + 9y = 4.
Solution:
3x + 9y = 4
−3𝑥 4
Y= +
9 9

JAY T. OLIVEROS, REE, RME


Engineering Mathematics

−𝑥 4
Y= +
3 9
Gradient of tangent = 3
Y = x³
𝑑𝑦
= 3𝑥² = 3
𝑑𝑥
x² = 1
x = 1 or -1
y = 1 or -1
Ans.: (1,1)

336. A boy wishes to use 100 feet of fencing


to enclose a rectangular garden. Determine the maximum possible area of his garden.
Solution:
2x + 2y = 100 and y = 50 – x
The area is f(x) = x (50 – x)
= -x² + 50x
−𝑏 −50
The maximum point is x = = = 25
2𝑎 −2
The maximum Area is f(25) = 25(50-25)
= 25(250)
=625 ft²

337. Find the equation of the tangent line to


the curve x³ + y³ = 9 at the given point (1,2).
Solution:
x³ + y³ = 9
𝑑 𝑑
(𝑥 3 + 𝑦 3 ) = (9)
𝑑𝑥 𝑑𝑥
𝐼
3𝑥² + 3𝑦²𝑦 = 0
−3𝑥 2
𝑦𝐼 =
−3𝑦 2
𝑥2
𝑦𝐼 = 2
𝑦
(1)²
𝑦𝐼 =
(2)2
1
𝑦𝐼 =
4
Y = mx + b
1
2 = (1) + 𝑏
4
-8 – 1 = b
-9 = b
1
y= 𝑥−9
4
-4y= x – 9
x + 4y = 9

JAY T. OLIVEROS, REE, RME


Engineering Mathematics

338. Find the area of the largest rectangle


whose base is on the x axis and whose upper two vertices line on the curve y = 12 - x².
Solution:
A=LxW
A(X) = 2x (12- x²)
= 24x – 2x³
𝐼( )
𝐴 𝑥 = 24 − 6𝑥²
24 – 6x² = 0
24 = 6x²
4 = x²
X=2
A = 2(2) * [12-(2)²]
A = 4*8
A = 32

339. Find the radius of the largest right


circular cylinder inscribed in a sphere of radius 5.
Solution:
V = 𝜋𝑟 2 (2ℎ)
Let the right circular cylinder have a radius r and height 2h. Cylinder has to fit inside a
sphere
r² = 5² - h²
V = 𝜋𝑟 2 (2ℎ)
= 𝜋(52 − ℎ2 )(2ℎ)
= 2𝜋(52 ℎ − ℎ3 )
𝑑𝑉
So, = 2𝜋(52 − 3ℎ2 )
𝑑ℎ
𝑑2 𝑉
= −12𝜋ℎ
𝑑ℎ2
𝑑𝑉 𝑑2 𝑉
At the maxima; = 0 𝑎𝑛𝑑 <0
𝑑ℎ 𝑑ℎ2
5 50
h= 𝑎𝑛𝑑 𝑟 2 =
√3 3
50
r=√
3
r = 4.08 units

340. a rectangular box open at the top is to


be constructed from a 12 x 12 inch piece of cardboard by cutting away equal squares from
the four corners and folding up the sides. Determine the size of the cutout that maximizes
the volume of the box.
Solution:
V = (12 – 2x)² x
V = 144x – 48x² + 4x³
Domain of function 0<x<6
V = 4x³ - 48x² + 144x
𝑉 𝐼 = 12𝑥 2 − 96𝑥 + 144
= 12 (x² - 8x + 12) = 0
= 12(x-6)(x-2) = 0

JAY T. OLIVEROS, REE, RME


Engineering Mathematics

x–6=0 x–2=0
x=6 x=2
if V(6) = 144(6) – 48(6)²+4(6)³
=864 – 1728 – 824
=0
If V(2) = 144(2) -48(2)²+4(2)³
V = 288-192+32
V = 128
Therefore the maximum volume is 128 when x = 2

341. Find dy/dx if y = 52𝑥+1 .


Solution:
𝑑
(52𝑥+1 )
𝑑𝑥
= 2ln(5).52𝑥+1
52𝑥+1 = 𝑒 (2𝑥+1)ln (5)
𝑑 𝑑
(𝑒 (2𝑥+1) ln(5) ((2𝑥 + 1) ln(5))
𝑑𝑥 𝑑𝑥
𝑑
((2𝑥 + 1) ln(5)) = 2 ln(5)
𝑑𝑥
= 𝑒 (2𝑥+1)ln (5) . 2ln (5)
= 𝑒 (2𝑥+2) ln(5) = 52𝑥+1
= 2ln(5).52𝑥+1
=(𝟓𝟐𝒙+𝟏 )𝒍𝒏𝟐𝟓

342. An athlete at point A on the shore of a


circular lake of radius 1 km wants to reach point B on the shore diametrically opposite A.
If he can row a boat 3 km/hr and jog 6 km/hr, at what angle with the diameter should he
row in order to reach B in the shortest possible time?

343. Find the area of the region above the x


axis bounded by the curve y = -x² + 4x – 3.
Solution:
y = -x² + 4x – 3
y = (3 - x) (x - 1)
3

∫(4𝑥 − 𝑥 2 − 3)𝑑𝑥
1
3 3 3

4 ∫ 𝑥𝑑𝑥 − ∫ 𝑥𝑑𝑥 − 3 ∫ 𝑑𝑥
1 1 1
3 3 3

4 ∫ 𝑥𝑑𝑥 − ∫ 𝑥²𝑑𝑥 − 3 ∫ 𝑑𝑥
1 1 1
𝑥23 𝑥3 3
4( )|1 − ( ) |1 − 3(𝑥) |13
2 3
9 1 27 1
= 4( − ) − ( − ) − 3(3 − 1)
2 2 3 3
26
= 16 – ( ) − 6
3

JAY T. OLIVEROS, REE, RME


Engineering Mathematics

4
= = 𝟏. 𝟑𝟑𝟑
3

344. Find the volume of the solid of


revolution formed by rotation the region bounded by the parabola y = x² and the lines y =
0 and x = 2 about the x axis.
Solution:
2
Vol = 𝜋 ∫0 𝑦² 𝑑𝑥
2
= 𝜋 ∫0 (𝑥 2 )2 𝑑𝑥
2
= 𝜋 ∫0 𝑥⁴ 𝑑𝑥
𝑥⁵
= 𝜋[ ]20
5
25
= 𝜋 [ ] − 𝜋(0)
5
= 6.4 𝜋
= 20.11 cu. units

345. A publisher estimates that in t months


after he introduces a new magazine, the circulation will be C(t) = 150t² + 400t + 7000
copies. If this prediction is correct, how fast will the circulation increase 6 months after the
magazine is introduced?
Solution:
C(t) = 150t² + 400t + 7000
=300t + 400
= (300)(6)+400
= 2200

346. What is the order and degree of the


differential equation y’’’ + xy’’ + 2y(y’)² + xy’ = 0.
Ans: Third order, first degree

347. A curve is defined by the condition that


at each of its points (x,y), its slope is equal to twice the sum of the coordinates of the point.
Express the condition by means of a differential equation.
𝒅𝒚
Ans.: = 𝟐𝒙 + 𝟐𝒚
𝒅𝒙

348. Find the first derivative of ln(cosx).


Solution:
𝑑 1 𝑑
ln (𝑐𝑜𝑠𝑥) = ∗
𝑑𝑥 𝑐𝑜𝑠𝑥 𝑑𝑥
1
= ∗ −𝑠𝑖𝑛𝑥
𝑐𝑜𝑠𝑥
− 𝑠𝑖𝑛𝑥
=
𝑐𝑜𝑠𝑥
= −𝒕𝒂𝒏𝒙

349. Find the number of equal parts into

JAY T. OLIVEROS, REE, RME


Engineering Mathematics

which a given number N must be divided as that their product will be a maximum.
Ans.: N/e

350. An object moves along the x-axis so


that its x-coordinate obeys the law x = 3t² + 8t + 1. Find the time when its velocity and
acceleration are the same.
Solution:
Differentiate the position:
V(t) = x’(t) = (3t² + 8t + 1)’ = 6t + 8
A(t) = v’(t) = (6t + 8) = 6
Equate v and a, we obtain
6t + 8 = 6
6t = 6 – 8
6t = -2
−1
t=
3
𝟏
ans.:
𝟑

351. Assuming that the earth is a perfect


sphere, with radius 4000 miles. The volume of ice at the north and south poles is estimated
to be 8,000,000 cubic miles. If this ice were melted and if the resulting water were
distributed uniformly over the globe, approximately what should be the depth of the added
water at any point on the earth?

352. Find the equation of the curve passing


through the point (3,2) and having slope 2x² - 5 at any point (x,y).
Solution:
𝑑𝑦
= 2𝑥² − 5
𝑑𝑥
Y = ∫(2𝑥 2 − 5)𝑑𝑥
2𝑥³
Y= − 5𝑥 + 𝑐
3
3y = 2x³ - 15x + c, c = -3
3y = 2x³ - 15x – 3
2x³ - 15x – 3y – 3 = 0

353. Find the centroid of the region


bounded by y = x², y = 0, and x = 1.
Solution:
1
A = ∫0 𝑥 2 𝑑𝑥
𝑥³
= [ ]10
3
1
=
3
1
1 𝑏 1
X=
𝐴
∫𝑎 𝑥 𝑓(𝑥 )𝑑𝑥 = 1
3
∫0 𝑥. 𝑥 2 𝑑𝑥
1
= 3∫0 𝑥 4 𝑑𝑥
3
= [ 𝑥⁴]10
4

JAY T. OLIVEROS, REE, RME


Engineering Mathematics

𝟑
=
𝟒

1 𝑏1 1 1
Ӯ= ∫𝑎 [𝑓(𝑥 )]2 𝑑𝑥 = 3 − ∫ (𝑥)²𝑑𝑥
𝐴 2 2 0

3 1 4
Ӯ = ∫0 𝑥 𝑑𝑥
2
3 𝑥5 1
=[ ∗ ]
2 5 0
3 15
= ∗
2 5
𝟑
=
𝟏𝟎

354. Find the point of inflection of the curve


x³ - 3x² - x + 7.
Solution
𝑑²𝑓
𝑑𝑥 2
f(x) = x³ - 3x² - x + 7
𝑑𝑓 𝑑2 𝑓
= 3𝑥² − 6𝑥 − 1 𝑎𝑛𝑑 = 6𝑥 − 6
𝑑𝑥 𝑑𝑥 2
𝑑²𝑓
is 0 when 6x – 6 - 0 or x = 1 and y = 4
𝑑𝑥²
For f(1) = 1³ - 3(1)² - 1 + 7 = 4
Thus the inflection point is (1,4)

355. Find two numbers whose sum is 36 if


the product of one by the square of the other is a maximum.
Solution:
Y = x²(36 – x)
= 36x² - x³
𝐼
𝑦 = 72 𝑥 − 3𝑥²
= 3x (24 – x) = 0
X – 24 = 0
X = 24
36 – 24 = 12
Ans: 12, 24

356. Find the minimum distance from the


curve y = 2 square root of 2x to the?

357. Divide 60 into3 parts so that the


product of the three parts will be the maximum. Find the product.
Solution:
60
= 20
3
(20)(20)(20) = 8000

JAY T. OLIVEROS, REE, RME


Engineering Mathematics

358. A particle moves along a path whose


parametric equations are x = t³ and y = 2t². What is the acceleration of that particle when
t = 5 seconds?

359. Find the area bounded by the curve 5y²


= 16x and the curve y² = 8x – 24.
Solution:
5y² = 16x
y² = 8x – 24

Solutions: (5,-4); (5, 4)


So our integrals is
𝑥2
4 +3 4 𝑦2 5
∫−4 𝑑𝑦 ∫ 8
5 2 𝑑𝑥 = ∫−4 ( + 3 − 𝑦 2 ) dy
𝑦 8 16
16

4 3
= ∫−4 (3 − 𝑦 2 ) dy
16
1
= (3y - 𝑦³) |4−4
16
= 24-8
= 16

360. Find the area in the first quadrant


bounded by the parabola y² = 4x and the line x = 3 and x = 1.
Solution:
y² = 4x
y = 2√𝑥
3 3
A = ∫1 𝑦𝑑𝑥 = 2 ∫1 √𝑥 𝑑𝑥
2𝑥 3/2 4
= = 𝑥 3/2 |13
3/2 3
3
4
= [(3)2 − (1)3/2 ]
3
A = 5.595 unit²

361. Find the area enclosed by the curve x² +


8y + 16 = 0, the line x = 4 and the coordinate axes.
Solution:
x² + 8y + 16 = 0
x² = -8y – 16
x² = -8(y+2)
thus vertex is at (0, -2)

4
A = ∫0 −𝑦𝑑𝑥
x² + 8y + 16 = 0
-8y = x² + 16
𝑥²
-y = +2
8

JAY T. OLIVEROS, REE, RME


Engineering Mathematics

4 𝑥2 𝑥³
A = ∫0 ( + 2) 𝑑𝑥 = + 2𝑥 |40
8 3(8)
(4)³
A= + 2(4) = 𝟏𝟎. 𝟔𝟕
24

362. Find the volume of the solid formed by


rotating the curve 4x² + 9y² = 36 about the line 4x + 3y – 20 = 0.
Solution:

Radius of latter
Circle r = 4
Semi major axis = 3
Semi minor axis = 2
V = 𝜋𝑎𝑏 𝑥 2𝜋𝑟
V = 𝜋(3)(2)𝑥 2𝜋(4)
V = 48𝝅² 𝒐𝒓 𝟒𝟕𝟑. 𝟕𝟒

363. Determine the moment of inertia of a


rectangle 100 cm by 300 cm with respect to a line through its center of gravity and parallel
to the shorter side.
Solution:
100

300

ℎ 3
𝑏( )
2 𝑏ℎ ℎ 2
𝐼ℎ𝑎𝑙𝑓 = + .( )
12 2 4
𝑏ℎ³ 𝑏ℎ³
= +
96 32
𝑏ℎ3
=
34
For the whole section,
𝑏ℎ³
I = 2𝐼ℎ𝑎𝑙𝑓 =
12
100(150)3
I = 2( + 100 𝑥 150 𝑥 (75)²)
12
= 2(28125000 + 84375000) cm⁴
= 225000000 or 225 x 𝟏𝟎𝟔 𝒄𝒎⁴

364. Find the area of the region bounded by


y² = 8x and y = 2x.
Solution:
y² = 8x y = 2x

JAY T. OLIVEROS, REE, RME


Engineering Mathematics

(2x)² = 8x y = 2(2)
4x² = 8x y=4
X=2

2 2
A = ∫0 𝑦𝑑𝑥 = ∫0 (𝑦𝑝− 𝑦𝐿 )𝑑𝑥
2
= ∫0 4 (√8𝑥 − 2𝑥)𝑑𝑥
3/2
√8𝑥 2𝑥 2 2√8
= - = 𝑥 3/2 − 𝑥²|20
3/2 2 3
2√8
= (2)3/2 − 2(2)
3
A = 4/3

365. Two posts, one 8 ft. high and the other


12 ft high, stand 15 ft apart from each other. They are to be stayed by wires attached to a
single stake at ground level, the wires running to the tops of the posts. How far from the
shorter post should the stake be placed to use the least amount of wire?
Solution:
L₁ = √𝑥² + 8² = √𝑥 2 + 64
L₂ = √(15 − 𝑥 )2 + 12² = √(15 − 𝑥 )2 − 144
Total length of wire
L = L₁ + L₂
L = √𝑥 2 + 64 + √(15 − 𝑥 )2 − 144
𝑑𝐿 2𝑥 2(15 − 𝑥)(−1)
= +
𝑑𝑥 2√𝑥² + 64 2√(15 − 𝑥 )2 + 144
𝑥 15 − 𝑥
=
2
√𝑥 + 64 √(15 − 𝑥 )2 + 144
x√(15 − 𝑥 )2 + 144 = (15 − 𝑥)√𝑥² + 64
x²[(15-x)²+144x²=x²(15-x)+64(15-x)²
144x² = 64(225-30x+x²)
9x² = 4(225-30x +x²)
5x² + 120x – 900 = 0
x² + 24x – 180 = 0
(x + 30) (x – 6) = 0
X = -30 x = 6
Ans.: 6 ft

366. At the maximum point, the second


derivative of the curve is
Ans.: Negative

367. Determine the curvature of the curve y²


= 16x at the point (4,8).
Solution:
y² = 16x
y² - 16x = 0

JAY T. OLIVEROS, REE, RME


Engineering Mathematics

2yyᶦ - 16 = 0
16 8
yᶦ = =
2𝑦 𝑦
𝑦(0)−8𝑦ᶦ
yᶦᶦ =
𝑦²
−8𝑦ᶦ
=
𝑦²
8
yᶦᶦy² = -8( )
𝑦
−64
yᶦᶦ =
𝑦³
8
yᶦ =
𝑦
yᶦ= 1
−64 −64 −1
yᶦᶦ = = =
𝑦³ (8)³ 8
|𝑦ᶦᶦ|
k=
[1+(𝑦ᶦ)²]3/2
−1
8
k=
[1+(1)²]3/2
k = -0.0042

368. Determine the value of the integral of


sin⁵3xdx from 0 to pi over 6.
Solution:
𝜋
6

∫ 𝑠𝑖𝑛5 3𝑥𝑑𝑥
0
Apply u-substitution: u = 3x
𝜋
1
=∫02 𝑠𝑖𝑛5 (𝑢) 𝑑𝑢
3
𝜋
1
= ∫0 𝑠𝑖𝑛5 (𝑢)𝑑𝑢
2
3
𝜋
1
= ∫02 (1 − 𝑐𝑜𝑠²(𝑢)2 𝑠𝑖𝑛(𝑢)𝑑𝑢
3
V = cos(u)
1 0
= ∫1 −1 + 2𝑣 2 − 𝑣 4 𝑑𝑣
3
1 1
= (− ∫0 −1 + 2𝑣 2 − 𝑣 4 𝑑𝑣)
3
1 1 1 1
= (−(− ∫0 1𝑑𝑣 + ∫0 2𝑣 2 − ∫0 𝑣 4 𝑑𝑣 ))
3

1
= ∫0 1𝑑𝑣 = 1
1 2
= ∫0 2𝑣² 𝑑𝑣 =
3
1 1
= ∫0 𝑣 4 𝑑𝑣 =
5
1 2 1
= (− (−1 + − ))
3 3 5
𝟖
= 𝒐𝒓 𝟎. 𝟏𝟕𝟖
𝟒𝟓

369. A body moves such that its acceleration

JAY T. OLIVEROS, REE, RME


Engineering Mathematics

as a function of time is a = 2 + 12t, where “a” is in m/s². If its velocity after 1 s is 11 m/s,
find the distance traveled after 5s.

370. A runner and his coach are standing


together on a circular track of radius 100 meters. When the coach gives a signal, the
runner starts to run around the track at a speed of 10 m /s. How fast is the distance
between the runners has run ¼ of the way around the track?
Solution:
The (x,y) coordinates of the runnercan be writtenas
X = 100cos(wt)(in meters,m)
Y = 100sin(wt)
At y = 0 (x,y) = (100,0) and the runner is travelling perpendicular to the x-axis
His velocity therefore is
𝑑𝑦
= 100𝑤𝑐𝑜𝑠(𝑤𝑡) = 100𝑤 = 10𝑚/𝑠
𝑑𝑡
So, w = 0.1 (1/s, inverse seconds)
The distance from the trainer at (100,0) and the runner at x,y is given by
D² = (x – 100)² + y²
Differentiating
2D(dD/dt) = 2(x – 100)(dx/dt) + 2y(dy/dt)
When the runner has gone ¼ of the way, he hits the y-axis so x = 0 and D = 100√2
He is also traveling perpendicular to y-axis so dy/dt = 0
dD/dt = -100(dx/dt)/D
dx/dt = -100wsin(wt) = -100w and ¼ the way around (wt = pi/2)
dD/dt = 100(100w)/D = (100w)/√2
𝑚
10/√2 = 5√2 𝑜𝑟 𝟕. 𝟎𝟕 𝐦/𝐬
𝑠

371. A telephone company has to run a line


from a point A on one side of a river to another point B that is on the other side, 30 km
down from the point opposite A. The river is uniformly 10 km wide. The company can run
the line along the shoreline to a point C then run the line under the river to b. The cost of
laying the line along the shore is P5000 per km, and the cost of laying it under water is
P12, 000 per km. Where the point C should be located to minimize the cost?

372. The height of the projectile thrown

JAY T. OLIVEROS, REE, RME


Engineering Mathematics

vertically at any given time is define by equation h(t) = -16t² + 256 ft. What is the maximum
height reach by the projectile?
Solution:
V = 0 at highest point
H(t) = -16t² + 256t
𝑑ℎ(𝑡)
V= = −32𝑡 + 256
𝑑𝑡
At highest point v = 0
-32t + 256 = 0
256
t= = 8 𝑠𝑒𝑐
32
h(8) = -16(8)² + 256(8)
h = 1023 ft.

373. The density of the rod is the rate of


change of its mass with respect to its given length. A certain rod has length of 9 feet and
total mass of 24 slugs. If the mass of a section of the rod of length x from its left end is
proportional to the square root of this length, calculate the density of the rod 4 ft from its
left end.

374. It costs 0.05 x² + 6x + 100 dollars to


produce x pounds of soap. Because of quantity discounts, each pound sells for 12 – 0.15x
dollars. Calculate the marginal profit when 10 pounds of soap is produced.

375. Find the area of the region bounded by


y = x² - 5x + 6, the axis, and the vertical lines x = 0 and x = 4.
Solution:
1 4
∫ (𝑥 2 − 5𝑥 + 6)𝑑𝑥 − ∫ ( (𝑥 2 − 5𝑥 + 6)𝑑𝑥
0 1
1
23
∫ 𝑥 2 − 5𝑥 + 6𝑑𝑥 =
0 6
4
3
∫ 𝑥 2 − 5𝑥 + 6 𝑑𝑥 =
1 2
23 3 𝟕
= − =
6 2 𝟑

376. A police car is 20 ft away from a long


straight wall. Its beacon, rotating 1 revolution per second, shines a beam of light on the
wall. How fast is the beam moving when it is nearest to the police car?
Solution:
𝑑Ɵ 1 𝑥 2𝜋Rad
= = 2𝜋Rad/min
𝑑𝑡 1𝑚𝑖𝑛
𝑥
𝑡𝑎𝑛Ɵ =
20
𝑑𝑡𝑎𝑛Ɵ 1 𝑑𝑥
= 𝑥
𝑑𝑡 20 𝑑𝑡

JAY T. OLIVEROS, REE, RME


Engineering Mathematics

𝑑(𝑡𝑎𝑛Ɵ) 𝑑Ɵ 1 𝑑𝑥
𝑥 = 𝑥
𝑑𝑡 𝑑𝑡 20 𝑑𝑡
𝑑Ɵ 1 𝑑𝑥
sec²Ɵ x = 𝑥
𝑑𝑡 20 𝑑𝑡
𝑑Ɵ 1 𝑑𝑥
(1+tan²Ɵ) = 𝑥
𝑑𝑡 20 𝑑𝑡
1
When x = 1, tanƟ =
20
1 𝑑Ɵ 1 𝑑𝑥
(1+ ) = 𝑥
400 𝑑𝑡 20 𝑑𝑡
𝑑Ɵ
= 2𝜋
𝑑𝑡
401 1 𝑑𝑥
𝑥 2𝜋 = 𝑥
400 20 𝑑𝑡
dx
20 x 2𝜋 =
dt
𝒅𝒙
40𝝅 =
𝒅𝒕

377. Find area of the largest rectangle that


can be inscribed in an equilateral triangle of side 20.
Solution:

20 20

10 10

Note: the area of the largest rectangle inscribed in an equilateral triangle is ½ the area of
that triangle.
h =10tan60
h = 10√3
A = ½ bh
A = ½ (20)(10√3)
A = 173.20

Area of largest rectangle inscribed = Area of equilateral triangle/2


173.20
= = 𝟖𝟔. 𝟔 𝐨𝐫 𝟓𝟎√𝟑
2

378. A hole of 2 radius is drilled through the


axis of a sphere of radius 3. Compute the volume of the remaining part.
Solution:
4
V = 𝜋(R2 − 𝑟 2 )3/2
3
4
V = 𝜋((3)2 − (2)2 )3/2
3
V = 46.83

379. Find the maximum area of a rectangle

JAY T. OLIVEROS, REE, RME


Engineering Mathematics

circumscribed about a fixed rectangle of length 8 and width 4.


Solution:
1 1
Maximum Area = (8 x 4) + 2 ( (8𝑥4) + (4𝑥2)
2 2
=72

380. A trough filled with liquid is 2 m long


and has a cross section of an isosceles trapezoid 30 cm wide of 50 cm. If the through
leaks water at the rate 2000 cm³/min, how fast is the water level decreasing when the
water is 20 cm deep.

381. Determine the area of the region


bounded by the curve y = x³ - 4x² + 3x and the axis, from x = 0 to x = 3.
Solution:
1 3
A = ∫0 𝑓 (𝑥 )𝑑𝑥 − ∫1 𝑓(𝑥 )𝑑𝑥
1 3
= ∫0 (𝑥 3 − 4𝑥 2 + 3𝑥 )𝑑𝑥 − ∫1 (𝑥 3 − 4𝑥 2 + 3𝑥 )𝑑𝑥
𝟑𝟕
A=
𝟏𝟐

382. Find the volume of a solid formed by


rotating the area bounded by y = x², y = 8 - x² and the y axis about the x axis.
Solution:
y = x²
y = 8-x²
x² = 8-x²
2x² = 8
x = +2
𝑏
V =𝜋 ∫𝑎 ([𝑓 (𝑥 )]2 − [𝑔(𝑥 )]2 )𝑑𝑥
2
= 𝜋 ∫−2[(8 − 𝑥²) ² − (𝑥 2 )²]𝑑𝑥
2
= 𝜋 ∫−2[(64 − 16𝑥 2 + 𝑥 4 ) − (𝑥 4 )]𝑑𝑥
2
= 𝜋 ∫−2(64 − 16𝑥 2 )𝑑𝑥
2
=16𝜋 ∫−2(4 − 𝑥 2 )𝑑𝑥
𝑥3
= 16 𝜋(4𝑥 − )|2
3 −2
8 2
= 16 𝜋(8 - )|
3 −2
8 8
= 16 𝜋 (8 − ) − (−8 + )
3 3
32
= 16 𝜋 .
3
512
𝜋⁄
= 13
2

383. The price p of beans, in dollars per

JAY T. OLIVEROS, REE, RME


Engineering Mathematics

basket, and the daily supply x, in thousands of basket, are related by equation px + 6x +
7p = 5950. If the supply is decreasing at the rate of 2000 basket per day, what is the rate
of change of daily basket price of beans when 100,000 baskets are available?

384. A flying kite is 100 m above the ground,


moving in a horizontal direction at a rate of 10 m/s. How fast is the angle between the
string and the horizontal changing when there is 300 m of string out?
Solution:
x

100m
300m

𝑑𝑥
= 10 𝑚/𝑠
𝑑𝑡
Get an eq. related to the x andƟ
𝑥
= 𝑐𝑜𝑡Ɵ
100
𝑑𝑥 𝑑Ɵ
0.01 = −𝑐𝑠𝑐²Ɵ
𝑑𝑡 𝑑𝑡
𝑑𝑥
𝑑Ɵ 0.01
= 𝑑𝑡
𝑑𝑡 −𝑐𝑠𝑐²Ɵ

0.01(10)
= 300
−( )²
100

𝟏 𝟏
=- 𝒐𝒓 𝒓𝒂𝒅/𝒔𝒆𝒄
𝟗𝟎 𝟗𝟎

385. If functions f and g have domains Df


and Dg respectively, then the domain of f/g is given by
Ans.: the intersection of Df and Dg
Without the zeros of function g

386. Let the closed interval [a,b] be the


domain of function f. The domain of f(x – 3) is given by
Ans.: the closed interval [a + 3, b + 3]

387. Let the interval (a, + infinity) be the


range of function f. The range of f (x) – 4 is given by
Ans.: The interval (a -4, + infinity)

388. If functions f(x) and g(x) are continuous


everywhere then

JAY T. OLIVEROS, REE, RME


Engineering Mathematics

Ans.: (f/g)(x) is also continuous everywhere except at the zeros of g(x)

389. If functions f(x) and g(x) are continuous


everywhere and f(1) = 2, f(3) = -4, f(4) = 8, g(0) = 4, g(3) = -6 and g(7) = 0 then lim(f +
g)(x) as approaches 3 is equal to

390. If f(x) and g(x) are such that lim f(x) as


x→a = + infinity and lim g(x) as x→a = 0, then
Ans.: lim [f(x). g(x)] as x →a may be + infinity or – infinity

391. A critical number c of a function f is a


number in domain of f such that
Ans.: f’(c) = 0
Or f’(c) is undefined

392. The values of parameter a for which


function f defined by f(x) = x³ + ax² + 3x has two distinct critical numbers are in the interval.
Ans.:f(x) = x³ + ax² + 3x
The derivative of f is given by
F(x) = 3x² + 3ax + 3
The critical numbers may be found by solving
f’(x) = 3x² + 3ax + 3 = 0
the discriminant D of the above equation is given by
D = (2a)² - 5(3)(3) = 4a² - 36
D is positive and the quadratic equation has 2 distinct solutions a in interval
(-infinity, -3) U (3,+infinity)
Therefore, none of the choices is correct

393. If f(x) has one critical point at x = c, then


Ans.: function f(x – a) has one critical point at x = c + a or f(kx) has a critical point
at x = c/k

394. The values of parameter a for which


function f defined by f(x) = 3x³ + ax² + 3 has two distinct critical numbers are in the interval

395. If f(x) = x³ - 3x² + x and g is the inverse


of f, then g’(3) is equal
Solution:
F(x) = x³ - 3x² + x
F’(x) = 3x² - 6x + 1
g is inverse of f so,
1
g’(x) = 2
3𝑥 −6𝑥+1
1
g’(3) =
3(3)−6(3)+1
𝟏
g’(3) =
𝟏𝟎

396. If functions f and g are such that f(x) =

JAY T. OLIVEROS, REE, RME


Engineering Mathematics

g(x) + k where k is a constant, then


Ans.: f ‘(x) = g ‘ (x)

397. If f(x) = g(u) and u = u(x) then


Ans.: f ‘(x) = g ‘ (u) . u ‘ (x)

398. lim [𝑒 𝑥 − 1]/𝑥 as x approaches 0 is


equal to
Ans. 1

399. If f(x) is a differentiable function such


that f’(0) = 2, f’(2) = -3 and f’(5) = 7 then the limit lim [f(x) – f(4)]/(x-4) as x approaches 4 is
equal to
Ans.: 4

400. If f(x) and g(x) are differentiable


functions such that f’(x) = 3x and g’(x) = 2x² then the limit lim [(f(x) + g(x)) – (f(1) + g(1))]/(x-
1) as x approaches 1 is equal to
Ans.: 5

JAY T. OLIVEROS, REE, RME

You might also like